2ちゃんねる ■掲示板に戻る■ 全部 1- 最新50    

■ このスレッドは過去ログ倉庫に格納されています

スレッドを立てるまでもない質問スレッド Part 317

422 :えワ:2015/12/05(土) 02:56:12.10 ID:iB4eC+Sx.net
>>421
「政史」君、いらっしゃい。
SEALDsに殺人予告をしたのは、君だったんだね。

423 :えワ:2015/12/05(土) 03:01:33.79 ID:iB4eC+Sx.net
>>421
君が言ってたこの情報は正しいの?
〒168-0063 東京都杉並区和泉一丁目5-13 チビ

424 :えワ:2015/12/05(土) 03:04:03.35 ID:iB4eC+Sx.net
>>421
君が言ってたこの情報は正しいの?
〒194-0002 東京都町田市南つくし野2丁目16-1 ぱよぱよち〜ん

425 :あぼーん:あぼーん
あぼーん

426 :あぼーん:あぼーん
あぼーん

427 :名無しさん@英語勉強中:2015/12/05(土) 06:26:09.10 ID:piPai6QO.net
>>413
○○首相は国際会議の場で、他の首脳陣から最も孤立し存在感がない
  Prime Minister BBB stays the most aloof from his counterparts,
  failing to stand out at international conferences.

428 :名無しさん@英語勉強中:2015/12/05(土) 06:31:45.56 ID:htBfedvY.net
The cherry blossom may well bloom early.

この文ですが、wellはどういうことを指してるのでしょうか?

429 :名無しさん@英語勉強中:2015/12/05(土) 06:33:32.02 ID:htBfedvY.net
The day of the party is not fixed yet.

この場合、まだ〜きまってない。ですが、
まだ、だとstillが使われてもいい気がするのですが、stillの否定形が not- yetという考えであってますでしょうか?

430 :名無しさん@英語勉強中:2015/12/05(土) 06:34:56.26 ID:piPai6QO.net
>>411
以下の出典は、すべて Corpus of Contemporary American English。

(1) Howard had warned him, 【told him how to address her
  and how to act in her presence】. (小説より)
(2) Later, his first and only lover had 【told him how】 impressed she was
   that he had good judgment for someone his age, (小説より)
(3) She never 【told him how】 she was feeling. (会話より)

431 :名無しさん@英語勉強中:2015/12/05(土) 06:42:47.34 ID:piPai6QO.net
>>428
You 【may well】 be surprised at the news.
   君がそのニュースを聞いて驚くのも当然だ。
     (You have good reason to be....
     It is natural that you should be....)
       (ジーニアス英和、第5版)

432 :名無しさん@英語勉強中:2015/12/05(土) 06:49:27.52 ID:htBfedvY.net
>>431
mayを強調するためにwellがついてるのでしょうか?

433 :名無しさん@英語勉強中:2015/12/05(土) 07:44:30.50 ID:Ta3l1Qgb.net
It is very cold today nevertheless do you play tennis?

434 :名無しさん@英語勉強中:2015/12/05(土) 12:18:46.73 ID:Rl4+1Ncj.net
彼は再びアメリカと戦火を交える日本の姿に憧れているを

He admires Japan's figure that its Army fights with America again
のように訳してみましたが、姿はfigureなのですが、figureの内容を説明する際、
同格のthat説は使えるのでしょうか?

他にもっといい英訳があれば、教えてください。

435 :名無しさん@英語勉強中:2015/12/05(土) 12:24:58.06 ID:m4rXwp3f.net
>>434
He is dreaming of Japanese army will fight against America in the future again.

436 :名無しさん@英語勉強中:2015/12/05(土) 12:54:50.79 ID:piPai6QO.net
>>428
may well [do something] 二はもう一つ意味があり、今回の場合には次の
場合が適合するのだということに後で気づいた。

It 【may well】 rain before tonight. 晩までにたぶん雨になりそうだ
  (ジーニアス英和、第5版)

437 :名無しさん@英語勉強中:2015/12/05(土) 13:01:20.21 ID:piPai6QO.net
>>434
>>彼は再びアメリカと戦火を交える日本の姿に憧れている

  He is longing to see Japan at war with America again.

438 :名無しさん@英語勉強中:2015/12/05(土) 13:17:50.93 ID:piPai6QO.net
>>429
not fixed yet
  = not yet fixed
  = still unfixed
  = still not fixed

詳しくは、二種類くらいの辞書で "not yet" と still とを使った例文をじっくり
眺めて調べてください。

439 :えワ:2015/12/05(土) 14:12:35.38 ID:iB4eC+Sx.net
>>432
>mayを強調するため
may well で may より「確信の度合いが強い」というのが辞書の説明。
will との使い分けは、個人で決める事なんだろうね。

The schedule is fixed.
The schedule is fixing.
The schedule is not fixed.
文章はこれで足りるんで、still も yet も「強調」の意味を持っている。
状況によって意味が変わるけど、例えば
もう決まってなきゃいけないのに、まだ決まっていない。
「もう決まってなきゃいけないのに」が yet (単に、まだ、ではない)
設定中、ただ、色々もめてる。
「色々もめてる」が still

特に still はかなり強い単語なんで、使い方を確認した方がいい。
文脈しだいでは「すごいもめてる」辺りまでの意味を持ってる。

440 :名無しさん@英語勉強中:2015/12/05(土) 14:21:03.05 ID:JKizDFkI.net
公開中の "SECRET OF THEIR EYES" という映画で、ある種の単語の医学的な定義が、

moron・・・IQ=50〜69
imbecile・・・IQ=20〜49
idiot・・・IQ=〜19

だと語られていたけど、世間一般の感覚としても、グレード的には

IDIOT > IMBECIL > MORON

で合ってる?

441 :三年英太郎 ◆3CZBjOt3.Y :2015/12/05(土) 14:53:03.89 ID:92QvSqWD.net
嘘くさいなぁw
IQって真ん中が100なの知ってるの?
80くらいでも世間的は相当馬鹿なのに、50とか20とかマヂで池沼ですから!

442 :三年英太郎 ◆3CZBjOt3.Y :2015/12/05(土) 14:56:27.89 ID:92QvSqWD.net
と思ったらマヂだったっ!Σ(゚□゚;)

> The term arises from the Latin word imbecillus, meaning weak, or weak-minded.
> It included people with an IQ of 26?50, between "moron" (IQ of 51?70) and "idiot" (IQ of 0?25).[3]
https://en.wikipedia.org/wiki/Imbecile

443 :名無しさん@英語勉強中:2015/12/05(土) 14:58:07.75 ID:/HcBDR2p.net
最近関東で行方不明に鳴って80日くらいして見つかった障害児の子が
確かIQ19で一部で話題になってたはず

444 :三年英太郎 ◆3CZBjOt3.Y :2015/12/05(土) 15:02:35.98 ID:92QvSqWD.net
最近は使われないみたいね。
http://www.todayifoundout.com/index.php/2010/03/the-words-moron-imbecile-and-idiot-mean-different-things/

moron がもともとは希臘語から作った術語だったことはしらなんだ。
https://en.wikipedia.org/wiki/Moron_%28psychology%29

でもまあ、日常語としては「馬鹿」と「間抜け」と「阿呆」の順列を比べるくらい無意味なんじゃ?

445 :三年英太郎 ◆3CZBjOt3.Y :2015/12/05(土) 15:16:39.08 ID:92QvSqWD.net
IQ19って、そもそも鉛筆で何か書けるレベルなのかしらん?
どうやって計測するんだろう・・・
計測できないから19だったりして・・・

446 :名無しさん@英語勉強中:2015/12/05(土) 15:36:17.22 ID:z5AS+az7.net
ネットとかの無料IQテストは法則性の問題ばかりだよな

53 62 ?1 80
みたいなのとか

ルールを見つけるのが苦手な人がIQ低い人だと思う

447 :えワ:2015/12/05(土) 17:40:45.67 ID:iB4eC+Sx.net
>>440
日本人なら、こっちが分かりやすいだろうね、Webster の定義。

moron・・・・・ 8-to-12-year old (補助があれば、単純作業はこなせる)
imbecile・・・・4-to-8-year old (IQ=26〜50)
idiot・・・・・・・three-year old or younger (IQ=25以下)
あとは、「脳死(braub-dead)」

moron imbecile とも教育学、心理学の専門用語という扱いで、普通は idiot を使う。

銀行強盗に入った男、金をバッグに入れさせて、
強盗:「全部でいくらだ」
銀行員「5000万です」
強盗「今、領収書を書くから待ってろ」
で、住所とフルネームを書いた。 ← idiotically

448 :名無しさん@英語勉強中:2015/12/05(土) 18:33:22.04 ID:yL5NbYu/.net
braub www

449 :名無しさん@英語勉強中:2015/12/05(土) 21:53:14.45 ID:2adl02fb.net
「ド・レ・ミ・ファ・ソ・ラ・シ・ド」って英語でなんと言いますか?

450 :名無しさん@英語勉強中:2015/12/05(土) 22:02:42.59 ID:piPai6QO.net
>>449
下のリンク先のいろんな図を参照。

https://www.google.co.jp/search?q=do+re+mi+fa&newwindow=1&espv=2&biw=1242&bih=585&source=lnms&tbm=isch&sa=X&ved=0ahUKEwjzqLDY48TJAhWIipQKHbvoADUQ_AUIBigB

451 :えワ:2015/12/05(土) 22:14:07.63 ID:iB4eC+Sx.net
>>449
ひとまず、サウンドオブミュージックの「ドレミのうた」の歌詞。
「音楽用語 英語」でググると、色々ある。

Do Re Me Fa So La Ti
Let's see if I can make it easier

Doe, a deer, a female deer
Ray, a drop of golden sun
Me, a name I call myself
Far, a long long way to run
Sew, a needle pulling thread
La, a note to follow so
Tea, a drink with jam and bread
That will bring us back to Do, oh oh oh

Read more:?The Sound Of Music - Do-Re-Mi (Maria And The Children) Lyrics | MetroLyrics?

452 :名無しさん@英語勉強中:2015/12/05(土) 22:57:43.54 ID:OMyYesmV.net
柏野健次著「英語語法レファレンス」を購入したのですが、その中で
「seeは非段階的な語である。× I see him very much.」とあります。
非段階的とは程度の大小が考えられない単語だそうですが、ググると
I see him very much.なる表現が多数出てきます どういうことなんでしょうか?

453 :名無しさん@英語勉強中:2015/12/05(土) 23:14:02.98 ID:uZPtgK5E.net
as が後続してたり構文が違うじゃん
語法本読む前にまず大学入れよ

454 :名無しさん@英語勉強中:2015/12/06(日) 01:22:44.83 ID:GeVhzghT.net
Propelled by Jurgen Fritz's keyboard arsenal of grand piano, Hammond organ and Moog synth, the band were clearly versed in
the science of Brain Salad Surgery. But what they lacked in originality they made up for with mind-boggling chops.

But what they lacked in originality they made up for with mind-boggling chops.

これの構文がわかりません。よろしくお願いします。

455 :名無しさん@英語勉強中:2015/12/06(日) 02:18:12.70 ID:eaTSB0/S.net
>>452

456 :名無しさん@英語勉強中:2015/12/06(日) 02:25:43.52 ID:eaTSB0/S.net
>>454
(what they lacked in originality)O (they)S (made up for)V with mind-boggling chops.

457 :名無しさん@英語勉強中:2015/12/06(日) 02:26:54.80 ID:eaTSB0/S.net
>>452
"I see him very much"は144件しかヒットしないよ。その上使い方が違うからね。

458 :名無しさん@英語勉強中:2015/12/06(日) 04:26:58.47 ID:jHBpKk4r!.net
How do you think は間違いで、What do you think とwhatが正しい、というと、
How do you think という例はいくらでもあるぞ、となるが、
do you thinkの部分が単なる挿入だったりする。

そんなことを想起させるやりとりだな。

459 :えワ:2015/12/06(日) 04:47:38.37 ID:GzLrHibq.net
>>452
see も very much も意味が広い。
単語の並びが同じでも、「意味が同じ」ではないからね。
チンコピアスもそれが分かってなくて、意味も考えずに「同じ単語の並び」を貼るね。
中の人が同じだから、同じ馬鹿な考えをしてるのかな。

460 :名無しさん@英語勉強中:2015/12/06(日) 06:48:39.78 ID:TVqqH5i8.net
>>452
主語の I を省いて "see him very much" だけを COCA という corpus で検索すると、
次のような例文が出てくる。ただしこれらはすべて、not で否定されている。not で
否定されると see (a person) very much が許されるのだろうか?それについては
僕にはわからない。

(1) He got me this internship, but I probably won't ●see him very much●.
I'll be working for all the senators, especially the 50 Republicans, not just him.
  (2008, fiction, Corpus of Contemporary American English)
(2) I feel guilty about not going to ●see him very much●.
  I'm worried about it.
  (1992, magazine, Corpus of Contemporary American English)
(3) Don't bullshit me, George. I don't ●see you very much●, I don't want to waste the time.
  (2001, fiction, 同上の corpus)
(4) Of course we can, but Norman, I think you'll want to see them now.
  I think you'll want to see them very much now.
  (1994, fiction, 同上の corpus, ただしこの very much は want を修飾しているらしい)
(5) You don't ●see them very much●, dearie,
  (1990, news, 同上)
(続く)

461 :名無しさん@英語勉強中:2015/12/06(日) 06:49:17.85 ID:TVqqH5i8.net
>>452 への回答の後半

上記の (4) で書いたように、want to see him very much については、very much
が see ではなく、want を修飾しているらしいので、論外。そして want を would like
などに変えてある例も、同じ理由で論外。

次に、Google Books を見ると、次のような例がある。
(6) Mimir is the oracular giant-god in Norse mythology.
  ●I see him very much● as the god of oracles and prophecy.

しかし (6) では very much が see を修飾しているのではなく、as のあとを修飾している
らしいので、これも論外。これらの例をすべて除外し、not で否定されず、さらには
very much が want や like を修飾せずにあくまでも see を修飾していると思われる
例文が今のところ見つからない。もしそうなら、やはり柏野氏が言うことは本当だという
ことになる。現に僕自身も、読書していたり映画などを見ていて、"I see him very much."
やそれに似た言い回しに出くわしたことが、今のところはない。(そんな気がするだけかもしれないが。)

462 :名無しさん@英語勉強中:2015/12/06(日) 07:21:24.66 ID:TVqqH5i8.net
>>452
"I see him very much" ではなく "I see him a lot." またはそれに近い言い回しなら、
使われているように見える。

(7) His relationship with his son is " great. ●I see him a lot.● "
  He's grateful for that.
  (1993, magazine, Corpus of Contemporary American English)
(8) So I ●see him a lot● all during the day.
  (1991, spoken, 同上の corpus)
(9) I say, okay, if I can ●see him a lot●.
  (2003, fiction, 同上)
(10) Do they ●see him a lot●?
  (1999, spoken, ibid.)
(11) And did you ever go to confession? Did you ●see him a lot● after that?
  (1992, spoken, ibid.)

他にも "see him a lot" という例はありそうだ。次に、"I see him much." または
それに似た例を探してみる。すると not で否定されているものならたくさん見つかるが、
それ以外のものとしては少なく、主に疑問文である。

(12) " You ●see him much●? " # " At breaks. "
  (1999, fiction, ibid.)
(13) Come on! You ●see her much more● than I do. I mean, you work
  in the same building.
  (1999, fiction, ibid.)
(14) " You ●see her much●? " She shook her head. " She live around here? "
  (1990, fiction, ibid.)

もしかして、"see (OR saw) (somebody) (very) much (OR a lot)" という形は、
否定文や疑問文としては使えるが、平叙文としては使えない(らしい)ということだろうか?
僕にはよくはわからない。詳しい人がコメントしてくれるだろう。

463 :名無しさん@英語勉強中:2015/12/06(日) 09:01:26.89 ID:GeVhzghT.net
>>456
どうもありがとうございます。
強調のための倒置によって目的語が文頭に来ている解釈しました。

464 :名無しさん@英語勉強中:2015/12/06(日) 09:35:16.83 ID:8nzX1MSX.net
参考
http://languagelog.ldc.upenn.edu/nll/?p=2400

465 :名無しさん@英語勉強中:2015/12/06(日) 10:28:23.35 ID:Htb2yoHg.net
you lost weight.
you lost your weight.
この2つはどれくらい意味が違うのか教えてほしいです。

466 :名無しさん@英語勉強中:2015/12/06(日) 10:43:15.34 ID:s8wMGqjQ.net
重りと体重

467 :名無しさん@英語勉強中:2015/12/06(日) 10:57:23.58 ID:6ec2TUsp.net
アメリカ人と話していると、冠詞のaをエイと発音することがある人に出会うことが
それほどめずらしくない気がします、それで少し調べてみたのですが、以下のような
ことを言ってる人もいます。
https://www.youtube.com/watch?v=2LSTC7W7dtw
しかし、日本でいろんな英語学習書をあさってみた時に、コラム程度にもこういった
ことに触れてるものをみかけたことがありません。まあ、学習書のスタンスから
いうと、そうならざるを得ないということなんでしょうか?

468 :名無しさん@英語勉強中:2015/12/06(日) 11:02:03.69 ID:5LZCNvO5.net
お前の学習書ってのが何を示してるのか知らんが、その程度のことを見たことがないのはお前が勉強不足なだけ

469 :名無しさん@英語勉強中:2015/12/06(日) 11:12:16.01 ID:6ec2TUsp.net
>>467
そうですか?どういう本に書いてあるのか、おしえていただけないでしょうか?

470 :467:2015/12/06(日) 11:19:56.31 ID:6ec2TUsp.net
>>468
アンカーまちがえた。

471 :名無しさん@英語勉強中:2015/12/06(日) 11:23:51.72 ID:6ec2TUsp.net
なるほど、やっすい辞書にも普通にeiと書いてあるんですね。でも、教師から
教えてもらった記憶はないなあ。

472 :名無しさん@英語勉強中:2015/12/06(日) 11:28:14.53 ID:5LZCNvO5.net
自分の不勉強を教師のせいにしてるうちは駄目だろうな

473 :467:2015/12/06(日) 11:37:59.56 ID:6ec2TUsp.net
>>472
そうですか?ところで、aをエイと発音するのは強調表現とはかぎらず、その方が
発音しやすければ、そうしてもいいと書いてある本は知りませんか?

474 :452:2015/12/06(日) 14:48:42.81 ID:YpDRRWmo.net
>>462
詳細なご高察感謝いたします
語法の正誤を考えていくプロセスが非常に参考になりました

475 :名無しさん@英語勉強中:2015/12/06(日) 17:56:19.00 ID:iD2dH3rK.net
返金を一切求めず払いきるシチュエーションで、動詞としてデポジット使うのは適切?
京都市バスは『料金はトルボックスに払ってください。』って
意味のアナウンスでデポジットの命令形を使っている。
通訳会社に騙されてボラれているようにしか聞こえのだけど…。
小銭に両替してトルボックスに払えと言うのなら
キャストにプリーズつけるぐらいが一番誤解ない表現ではないですか?

476 :名無しさん@英語勉強中:2015/12/06(日) 19:40:37.63 ID:TVqqH5i8.net
>>475
どういう根拠で deposit がふさわしくないって言ってるのかわからない。せめて
該当するそのセンテンスの全文をきちんと書いてくれないと、何が何だかわからない。
deposit という動詞は、少なくとも次のような文脈では使われている。

(1) He decides to just dial and see if maybe the phone is free now, which
would be cool, but a little annoying after everything he's done to
gather the money. He dials the ten digits, presses the headset to his
ear: " One dollar seventy-five cents please. Please ●deposit● one dollar
and seventy-five for the first three minutes. " He shoots one quarter
after another into the slot, and the machine gulps them down. " Thank
you. You have three minutes.
   (2009, fiction, Corpus of Contemporary American English)

(2) " Thank you for making your selection, " the jukebox says in a
strange feminine voice. " A video is available. If you wish to view
it, please ●deposit● twenty-five cents and enter the code number... now. "
   (1994, magazine, ibid.)

(3) Paul walks up to a ticket machine. " Two tickets to Aoyama Icchome, "
he says in Japanese. " Welcome. Please deposit two hundred and eighty
yen, " the machine instructs him. He feeds in coins. " That was the
exact amount needed. You will change trains at Akasaka Mitsuke Station.
Thank you, " the machine says, churning out two tickets. "
   (1996, fiction, ibid.)

477 :名無しさん@英語勉強中:2015/12/06(日) 19:54:39.30 ID:8nzX1MSX.net
depositが預ける、預け金を支払うという意味が
多いからだろう。だけどここでは自販機に金を
入れるという語義で辞書にもあるから定型的かは
わからんが、意味がおかしいということはないだろうな

478 :えワ:2015/12/06(日) 19:57:32.54 ID:GzLrHibq.net
>>476
ibid が変だと気付いてね。
ただ、書き写すだけだと、そうなるんだろうね。

479 :名無しさん@英語勉強中:2015/12/06(日) 20:17:28.35 ID:m6uywdZF.net
If you ( ) ( ) pass the exam, you’ d better study hard.

もし試験に受かりたいのならしっかり勉強しなさい。

480 :名無しさん@英語勉強中:2015/12/06(日) 20:20:37.80 ID:rZBjAAM9.net
want to

481 :名無しさん@英語勉強中:2015/12/06(日) 21:06:16.71 ID:S2EkMDjP.net
>>479
are to

482 :名無しさん@英語勉強中:2015/12/06(日) 21:11:02.95 ID:WBmZJAfG.net
got to

483 :名無しさん@英語勉強中:2015/12/06(日) 21:15:31.95 ID:Htb2yoHg.net
>>466
Thank You!

484 :名無しさん@英語勉強中:2015/12/06(日) 22:04:09.25 ID:gDYzunaP.net
D'you mind goin' straight

みたいな「'」でアルファベットが省略された文章ってネイティブにとってどういう印象受けるものなんですか?
育ち悪い?

485 :名無しさん@英語勉強中:2015/12/06(日) 22:07:53.27 ID:IkbgxnrP.net
http://i.imgur.com/iAFhgr7.jpg
この画像のシルビアさんのコメントが外国人の方に大人気のようなのですが
どういう皮肉(?)になっているのですか?

マイケル・ジャクソンのスムースクリミナルと言う曲のコメント欄です

486 :名無しさん@英語勉強中:2015/12/06(日) 22:49:23.82 ID:Lkxl3V4O.net
Korea,Korean style-cultures(K-pop, Korean drama) and products made in Korea have far and away more popular than Japan and them of Japan almost all over the world

世界中のほとんどで、韓国や韓流ドラマや韓国製品は日本や日本の文化や製品(それら)よりもはるかに、人気にある
という文を英訳してみたのですが、Japanese style-cultures and products made in Japanを省略する際、them of Japanで通じるでしょうか?

487 :名無しさん@英語勉強中:2015/12/06(日) 22:52:48.08 ID:HzNa9FsO.net
I'm from ○○ prefecture in Japan.

って会話で使うのはおかしいですか?
in じゃなくて of の方が良いですか?

488 :名無しさん@英語勉強中:2015/12/06(日) 23:27:11.79 ID:2k3eXcyH.net
>>487
in でも of でもなくて、
カンマかもしれない。
例えば、
I'm from Kagawa Prefecture, Japan.
とか。

489 :名無しさん@英語勉強中:2015/12/06(日) 23:50:52.06 ID:HzNa9FsO.net
>>488
なるほど、ありがとう
口語的にはあんまり言わない表現ですかね?

490 :名無しさん@英語勉強中:2015/12/07(月) 00:04:07.20 ID:aFdsERKG.net
>>484
口語的、フツーのしゃべり言葉
フォーマルな用法ではない
ビジネス等の場では不適

491 :名無しさん@英語勉強中:2015/12/07(月) 00:07:00.62 ID:LhJ6tNpg.net
>>489
私はネイティブではないから、詳しいことはわからないです。
ごめんなさい。
いろんな似たような英語での表現の仕方があると思うけど、
話し言葉だから,冗長な表現はできるだけ避けられると思う。
日本の知名度?が低いところでは、
日本って何?どこ?って聞き返されるかもしれない。
間違っていたらごめんね。

492 :名無しさん@英語勉強中:2015/12/07(月) 01:14:26.71 ID:NWSC2Dk6.net
486のような誤認識の内容はうっとうしいのでやめてください
在日の韓国人なのだろうけど、先進国はもちろん他の途上国はそういうお国自慢は
あまりしないよ。前に歴史問題や政治的な内容であおってたのも同じ種類の人間だよ。
ここは英語スレなのだから

493 :えワ:2015/12/07(月) 02:19:36.46 ID:yyHGsJUA.net
>>484
これに限らないけど、「知らないよ」と「知らねぇよ」ぐらいの認識でいいと思う。
要は、使う「時と場合と人」を間違えなければ大丈夫だろう。
私は getta gonna も使わないけど、まぁ、通じればいいんだよ。


>>485
Unicorn::::::::::::::: 「アニーは父親にレイプされ殺された」 で、殺されたかどうか、が話題になってる。
SamMut::::::::::::::::「殺されてたら歌詞の中に書くだろう(死んだと書いていないから生きてる)」
Abby White::::::::「今でも生きてるんじゃない。」
Alexis Silvera:「アニー大丈夫か、しっかりしろ・・・彼女はしんでいた」
Andrea Henderson「あなたが youtube に書き込んだの」
あとは、歌詞を見て、君の立ち位置を決める事だね。

みんなが知ってる曲なのに、「私死んだと思ってた派」と「生きてる派」がいるという話かな。
「あしたのジョー」のラスト、「ジョーは死んでるのか」っていうネタと同じだと思う。


>>487
in よりは of が普通だと思う、と答えてから、
日本で英語を使っているなら、県名だけでいいだろうね。
海外なら、同じ緯度の都市を挙げると、どんな気候か理解してもらえると思う。

494 :名無しさん@英語勉強中:2015/12/07(月) 06:42:08.12 ID:roWtGE9a.net
>>475
アメリカのバスの当局が出している料金徴収システムについての公式文書でも、
こういう場合にきちんと deposit という動詞を使っている。

(1), (2) Collecting fares from passengers is an important responsibility
for the bus driver who must ensure that each passenger ●deposits● the
appropriate coins or bills into the farebox. This task is easier when
passengers know how much to ●deposit● into the farebox.

(3) The electronic fareboxes are equipped with a readout window that
displays the amount of money ●deposited● in the farebox by each passenger.

(4) Rule 8.03 - ●Depositing● Fares

(5) Once the passenger ●deposits● money into the farebox, it counts
the money and displays the amount to the driver on a digital display.

(6) However, the operator must still evaluate the ●deposit● to be sure it is
valid. Modern fareboxes measure only the size of coins (or the length
of dollar bills), so any slug or paper within a size tolerance will be
counted toward the fare by the farebox.

(続く)

495 :名無しさん@英語勉強中:2015/12/07(月) 06:42:39.98 ID:roWtGE9a.net
>>475 への回答の続き

(7) MARTA has indicated that
this type of equipment has been effective in reducing the number of
driver/passenger disputes over fare matters because the farebox reads
the quantity of the fare ●deposited●, the operator reads the amount
reported by the farebox and requests additional fare, if necessary.

(8) In addition to guiding the bus safely,
responding to questions, and other duties, the operator must check
the farebox on each fare ●deposit● and be sure that the passenger has
paid the correct fare.

以上 8 つの deposit の使用例は、すべて下記の公式文書からの引用。

Bus Transit Fare Collection
Practices
A Synthesis of Transit Practice
Transportation Research Board
National Research Council
(出版社名:NATIONAL ACADEMY PRESS Washington, D.C. 1997)
http://ntl.bts.gov/lib/8000/8000/8032/tsyn26.pdf

496 :名無しさん@英語勉強中:2015/12/07(月) 12:20:22.57 ID:JBBVpXx3.net
Lord of the ringsのBBCラジオ版のスクリプトからです。

ゴラムがナズグルに連れ去られるシーン
Gollum: No, precious! No, precious! Puts us down! Puts us down!


状況からすると「下ろしてくれ!」という意味だと思いますが、なぜputではなくputsなのでしょうか?

497 :三年英太郎 ◆3CZBjOt3.Y :2015/12/07(月) 12:22:38.96 ID:FZ8a5bWq.net
そもそもゴラムって、おかしな英語を喋るキャラでしょ。

498 :名無しさん@英語勉強中:2015/12/07(月) 17:31:16.83 ID://sMb69f.net
But the probe will not be able to collect as much data as initially intended
because it will be circling the planet at a much higher altitude than envisioned under its originally planned orbit time of 30 hours, which it was unable to achieve in December 2010



30 hours, which it was unable to achieve in December 2010ですが
なんで30 hours, which was unable to achieve in December 2010じゃないんでしょうか?
あとitは何を指しているんでしょうか?

499 :名無しさん@英語勉強中:2015/12/07(月) 17:48:42.93 ID:roWtGE9a.net
>>498
But ●the probe● will not be able to collect as much data as initially intended
because it (= the probe) will be circling the planet at a much higher altitude
than envisioned under its (= the probe's) originally planned orbit time of 30 hours,
which it (= the probe) was unable to achieve in December 2010

500 :名無しさん@英語勉強中:2015/12/07(月) 17:52:46.83 ID:roWtGE9a.net
>>498
もしもあなたの言う通りに
   30 hours, which was unable to
と書き始めるとしたら、
   30 hours, which was (または were) unable to be achieved....
と書かねばならない。なお、30 hours を一つのまとまりとして考えればそのあとは
was だけど、30 hours を複数と考えれば were となる。

501 :名無しさん@英語勉強中:2015/12/07(月) 19:06:49.38 ID:AbE6sQ+T.net
not A but Bについて
なぜAではなくBという訳になるのでしょうか
Aではなく、は分かります。notですし。
しかしbut Bが理解できません。
辞書通りならBも否定になりそうに思います。


but
【名】
しかしという言葉、異議◆【参考】No buts!文例
【接続】
しかし、ただし◆逆接文例
そうではなくて、〜ではなくて
【前】
〜以外の[に]、〜を除いて、〜をおいて

502 :名無しさん@英語勉強中:2015/12/07(月) 19:26:33.69 ID:wbiu6HJN.net
そういう表現なんだから仕方がない
いかにも言えそうな構文とは思わない?
それぞれの単語の辞書的語義の足し算が
そのまま文意になるわけないじゃん

503 :名無しさん@英語勉強中:2015/12/07(月) 19:38:26.36 ID:JBBVpXx3.net
>>497
つまり、単に変な英語ってことでしょうか?

504 :名無しさん@英語勉強中:2015/12/07(月) 19:41:40.13 ID:CJQOxIKU.net
否定と逆接続  Aじゃない、しかしB = AではなくB 

感情のこもった音声を聞いてみたら感覚がわかると思うよ。
ググてみなよ。 ビジネスパーソンがよく使う構文だよ。

505 :名無しさん@英語勉強中:2015/12/07(月) 19:45:54.17 ID:tFGmAtet.net
I was told that he is not home.
これって時制は一致させなくていいのでしょうか?

あと

Studying abroad to learn English will be like throwing your money down the drain.
金をどぶにすてるようなものだ。
という意味ですが、なぜ the drain なのでしょうか?この場合、ドブを指すdrainは実態がないので
a drainでもいけそうな気がするのですが、

506 :名無しさん@英語勉強中:2015/12/07(月) 20:17:46.90 ID://sMb69f.net
>>499 500
ありがとう

507 :名無しさん@英語勉強中:2015/12/07(月) 20:23:35.43 ID:468X+APf.net
>>505
I was told that he is not home. は時制をあわせるべきでしょう。
但し時制の一致には例外があります。東から日が昇るなど半永久的に事実で
ある場合は現在形です。 今回のケースで彼が家を巣立っていて、普段
に家に住んでいない半永久的な事を表現する時に使うかもしれません。
前後関係からチェックしてみてください。その様なニュアンスがある
かもしれませんね。

throwing your money down the drain. この場合のtheは感覚的なもので、話し手は
この例え話で、具体的なドブにお金を捨てるイメージ出来ているんですよ。だからそのドブ、
the drainに捨てると言っている。どれか1つのドブにすてるとはイメージしていないと思います。 
play the pianoなどと同じ感覚です。

508 :三年英太郎 ◆3CZBjOt3.Y :2015/12/07(月) 20:35:56.91 ID:FZ8a5bWq.net
> I was told that he is not home.

現在も not home の状態なら、このままでも良い。ロイヤルに多分でてる。

> the drain

W3に、「広く知られている自然環境・日常生活の事象に触れて」単数に the を付ける用法が載ってる。

509 :名無しさん@英語勉強中:2015/12/07(月) 20:42:38.03 ID:tFGmAtet.net
>>507
>>508

回答ありがとうございます。

the ですが、
いわゆる話者と聞き手が同じものを想起できてる場合に使われるケース=どれか1つのドブにすてるとはイメージしていない
広く知られている自然環境・日常生活の事象
以外にもtheが使われるケースで思い浮かべられるものはありますでしょうか?

510 :三年英太郎 ◆3CZBjOt3.Y :2015/12/07(月) 20:49:38.45 ID:FZ8a5bWq.net
>>509
青ロイヤル持ってるなら、
慣用的に定冠詞が取られる語句のリストが出てるよ。

511 :三年英太郎 ◆3CZBjOt3.Y :2015/12/07(月) 20:52:06.13 ID:FZ8a5bWq.net
>>503
https://en.wikipedia.org/wiki/Gollum#Speech

512 :名無しさん@英語勉強中:2015/12/07(月) 21:09:36.12 ID:tFGmAtet.net
>>510
慣用的に定冠詞がとられる名詞
もしくは、
話者と聞き手が同じものを想起できてる場合に使われるケース=どれか1つのドブにすてるとはイメージしていない

の二つがtheが使われるケースと考えていいのでしょうか?

513 :三年英太郎 ◆3CZBjOt3.Y :2015/12/07(月) 21:26:42.35 ID:FZ8a5bWq.net
冠詞はそんなに簡単ぢゃないので、文法書買って読んで。

> 話者と聞き手が同じものを想起できてる場合

間違い。
聞き手も「あーあれね!」と想起できると、*話し手が思っている*場合。
聞き手が実際に知っているかどうかは、問題でゎない。

514 :名無しさん@英語勉強中:2015/12/07(月) 21:28:25.07 ID:tFGmAtet.net
>>513
久野の謎解きを読みましたが、自分としては
この2点なのかな。という印象です。
ありがとうございました。

515 :名無しさん@英語勉強中:2015/12/07(月) 22:15:17.46 ID:wbiu6HJN.net
英語冠詞大講座
こんなの出るらしいぞw

516 :名無しさん@英語勉強中:2015/12/07(月) 23:19:00.26 ID:eecR44YO.net
>>501
http://www2u.biglobe.ne.jp/~horumons/iitai.html
これのbutを見るとわかる
他もとても面白い

517 :名無しさん@英語勉強中:2015/12/08(火) 09:39:26.48 ID:uPy3K8XH.net
https://twitter.com/BabyAnimaI/status/672502434577653760
http://pbs.twimg.com/media/CVU1ZqFWsAAnhb6.jpg

この張り紙の意味がわかりません

518 :名無しさん@英語勉強中:2015/12/08(火) 10:27:52.76 ID:3qb8Rv33.net
>>517
Pros and cons of boys:
Con: They're dicks.
Pro: Their dicks.

間違っててもいいから、自分なりにきちんと解釈してみて、
ここで詳細に説明してみてください。
そうしたら、あなたのわかっていない部分をみんなが教えてくれるでしょう。

519 :えワ:2015/12/08(火) 11:20:01.55 ID:I3e1qOMN.net
>>517
Pros and cons of boys:
お題「少年ならどっち?」

ヒント
Your welcome

520 :名無しさん@英語勉強中:2015/12/08(火) 12:04:15.22 ID:uPy3K8XH.net
Pros and cons
がわからないのでぐぐったくらいのレベルで…


長所 彼らはdicks
短所 彼らのdicks

dickがチンコって意味があるくらいしかわかりません…

521 :名無しさん@英語勉強中:2015/12/08(火) 12:50:50.59 ID:3qb8Rv33.net
>>517
(1) Pros and cons of boys:
  男の子たちの長所と短所:
  ★"a pro" は、「よい点」という意味の名詞。"a con" は、「悪い点」という意味の名詞。
  学習辞典には、"the pros and cons of...." という形で意味が載っている。

(2) Con: They're dicks.
  短所:あの人たちは、馬鹿です。
  ★They're = They are だけど、次の (3) の Their とまったく発音が同じ。

(3) Pro: Their dicks.
  長所:あの人たちのおチンチン。

つまり、(2) も (3) もまったく同じく [ðɛə(r) diks] と発音しているけど、
意味が違っている。そこが面白い。dick(s) という同じ発音、同じ綴りの
言葉をまったく違う意味で使っている。女の子あるいはゲイの男にとって、
男の子の取り柄は「おチンチン」だけど、同時に男の子は「ちんぽ野郎」
つまり「バカ」でもあるのだということ。これは、男性自身が自戒を込めて
書いているに違いないけど。まあ、どのように解釈しても面白い。

522 :名無しさん@英語勉強中:2015/12/08(火) 12:57:31.18 ID:3qb8Rv33.net
>>517
フランス人がこの英語を読むと、さらにジョークが加わって面白く感じられるに違いない。
というのも、フランス語では con が「女性器」とか「バカ者」という意味になるからだ。
フランス語で、「なんていう馬鹿な奴だ!」と罵りたいとき、
   Mais quel con! --- [mɛ kɛl kɔ̃]
と言う。そういう台詞を映画で聞いたことがよくある。

523 :名無しさん@英語勉強中:2015/12/08(火) 13:06:00.01 ID:uPy3K8XH.net
なんか込み入ったジョークの質問を初心者レベルの人間がして答え甲斐がなかったかもしれなしですが
丁寧な説明してくださってありがとうございます。
やたら質問しないで英語の勉強精進いたします。
(日本語も怪しいが)

524 :えワ:2015/12/08(火) 13:44:42.73 ID:I3e1qOMN.net
>>523
英語の同音異義語、で、意味はどっちが正しいか。
耳で聞いて同じでも、文法を考えると正しい意味が取れる、というのが題意。
con(×)彼らは刑事
pro(○)かれらのちんちん
答え)少年は刑事ではない。

これは、出題の意図、解き方
The distinction between "you're" and "your" might not look like much to some,
but confusing the two can deeply annoy those of us who value good grammar.
Following the rules of grammar makes your texts clearer and more respectable.

Learning the difference between homophones like "horde" and "hoard"
will also help you stay out of trouble with the grammar police.
We suggest checking out this list of grammar puns so you know how to stay on their good side !

同音の you're と your は意味が違う。
このルールを知っていると、意味が明確な文章を書けるようになる。
"horde" と "hoard" は同音ながら、文法を考えると、間違える事はない。
この後の例も見ておくと、正しい解釈が出来るようになる。

例えば、カンマの例
Eat kid (子供を食べよう) → Eat, kid (坊や、食え)

>>522 が考えるほど凝った質問ではないよ。

525 :名無しさん@英語勉強中:2015/12/08(火) 21:47:08.87 ID:g80lPG7/.net
I want something to drink.

I want to get something to drink.

I want to drink something.


これ全部文法上は問題ないですか?

526 :名無しさん@英語勉強中:2015/12/09(水) 02:26:33.72 ID:DfmdiBhl.net
ある文章から言葉を引用する際、文末にその書き表した作者の名前を

by 〜 と表記するのは不自然でしょうか?

527 :名無しさん@英語勉強中:2015/12/09(水) 02:37:36.14 ID:PszOjK4O.net
>>526
どういう目的で表記しようとしているの?
論文か何か?それとも違う目的?
もっと具体的に聞いた方が良いと思うよ。

528 :名無しさん@英語勉強中:2015/12/09(水) 02:48:17.84 ID:DfmdiBhl.net
SNS(ツイッター)で紹介する用途です。

529 :えワ:2015/12/09(水) 03:03:28.39 ID:vNxagDIP.net
>>528
「論文の書き方 引用」でググって、基礎知識を持った方がいいね。
それで、自分で判断出来ると思う。

私は日本語でも英語でも、「誰々の言葉ですが、」「誰々の言によれば」と名前を前に出す。

530 :名無しさん@英語勉強中:2015/12/09(水) 06:39:34.11 ID:1t8kLrg1.net
>>526
有名な言葉を引用してそのあとにその言葉を最初に書いた(話した)人の名前を書く
方法については、たぶんいろいろあるのだろうけど、すんなりと見つかった例を
3種類、紹介する。

(1) 一つ目のやり方
“Don't cry because it's over, smile because it happened.”
— Dr. Seuss
   https://www.goodreads.com/quotes
これは、人名の前に "M dash" つまり長いダッシュ(横棒)を入れるやり方。
Windows 上でなら、ALT key を押しながら右側に 3列 x 3行の形で並んでいるテンキー
(上に横に一列に並んでいるキーではない)にて 0151 をタイプすると出てくる。
この M dash が使えないときには、ただのハイフン( - )を使うことも許されているらしい。

(2) 二番目のやり方
Try to be a rainbow in someone's cloud.
Maya Angelou
  http://www.brainyquote.com/quotes/topics/topic_inspirational.html

ただし、上記のリンク先を見てわかるように、引用された言葉と人名とでは、色を変えてある。
Twitter では色を変えられないので、(1) のように M dash またはハイフンをつけて
おいた方が親切だろう。

(3) 3つ目のやり方
You miss 100 percent of the shots you never take.
—Wayne Gretzky
https://litemind.com/best-famous-quotes/

531 :名無しさん@英語勉強中:2015/12/09(水) 06:45:04.15 ID:1t8kLrg1.net
>>526
(1) では、引用された言葉に引用符(つまり " " という符号)がついて、そのあとに
スペースをつけたあとに M dash をつけている。
(2) では、引用符も M dash もない。その代り、人名を別の色で表示している。
(3) では、引用符がなく、そのあとに M dash をつけ、そのあとにスペースなしで
人名をつけている。

この他にもいろんな引用の仕方はあるだろうけど、自分で確かめてみたければ、
famous quotes というキーワードで Google 検索すれば、面白そうなサイトが
たくさん出てくる。
   https://www.google.co.jp/search?q=famous+quotes&oq=famous+quotes&aqs=chrome.0.69i59j0l5.2201j0j4&sourceid=chrome&es_sm=93&ie=UTF-8

532 :名無しさん@英語勉強中:2015/12/09(水) 06:49:13.47 ID:1t8kLrg1.net
>>526
M dash(長い横棒)の代わりにハイフンでもいいらしいと僕は言ったが、
次のリンク先で確かにハイフンらしきもので代用している。

   "Moral indignation is jealousy with a halo."
   - H. G. Wells (1866-1946)
      http://www.cs.virginia.edu/~robins/quotes.html

533 :名無しさん@英語勉強中:2015/12/09(水) 09:28:37.23 ID:1t8kLrg1.net
>>526
有名な短い言葉のあとに "by Tom Brown" のように続けるのは、今のところは
やめた方がいいと思う。僕の記憶では、そういう状況を見たことがないような
気がする。

ただし、昔の紙の雑誌の中での署名記事では、たとえば昔のアメリカの
TIME magazine の紙版においては、数ページの長い記事のあとに

   Reported by Tom Brown,
   a special correspondent in Baghdad

とか何とか書いていたような気がする。(ただしこれについては、大昔のことなので
はっきりとは覚えていない。基本的に僕は新聞雑誌に興味がないので、あまり
詳しくはないのだ。)

今のネット版の TIME, Newsweek, New York Times
などではどうなっているのかザアッと見てみたけど、記事の最後に by Tom Brown
のような形で署名しているのではなく、別の形で投稿者の名前が紹介されている。

534 :名無しさん@英語勉強中:2015/12/09(水) 11:16:22.11 ID:Curacv8G.net
A dart gets thrown away.
これは
投げ矢が投げられるのを避ける
といういみなのかなのか
投げ矢を投げ捨てる
という意味なのかわかりません
教えてください

535 :名無しさん@英語勉強中:2015/12/09(水) 12:36:10.71 ID:LXGdq0Xg.net
目標・フレンズやsexandthecityのような日常会話系ドラマを聞き取れるようになりたい
ネイティブと普通に会話したい

相談・勉強法
英会話教室通ってネイティブと話す勉強法が良いかなと思いますがどうでしょうか?

今はドラマを内容覚えたら、字幕無しで聞く
ニュースやyoutube聞く
洋楽聴いて歌詞を書き出す をやってます。

仕事で使う、TOEIC英検受ける予定はありません。

536 :名無しさん@英語勉強中:2015/12/09(水) 13:08:03.58 ID:1t8kLrg1.net
>>534
>>A dart gets thrown away.
  投げ矢は、投げ捨てられる。

このような "get + 過去分詞" の意味合いについては、学習英和辞典にも載っているので、
ぜひその解説を読んでみてください。

>>535
(1) ネイティブと会話できる能力
(2) アメリカのテレビドラマを字幕なしで楽しむ能力

この二つは、あまりにもレベルが違いすぎます。(1) の方が、あまりにも低い。
あなたは、まずは (1) ができるようにならないといけませんので、(2) は
しばらくは忘れた方がいいと思います。通常の会話さえできない人が、
とてもじゃないけど字幕なしでドラマを何百回も見たって、ほとんど意味はないと思います。
むしろ、まずは基礎英語、続基礎英語、テレビ・ラジオ英会話などの無料の英語番組
を見まくり、聞きまくり、すべて覚えてしまうことです。もし文法があやふやなんだったら、
まずは基礎英語と続基礎英語をマスターすることです。

537 :名無しさん@英語勉強中:2015/12/09(水) 15:51:18.37 ID:0ou3v9bB.net
ちょっと古い歌なんですが、
槇原敬之のcowboyの歌詞
ttp://www.kasi-time.com/item-5500.html

の、

>But I never could fit in, it just wasn't me

>Searching for my dreams, in some far away place
>with this quest and my will to survive, to keep me alive

>※Hey hey show me you're a Cowboy, gotta do it now boy

の三箇所の意味と文法がわからなかったのでよろしければお教え願えませんか?

538 :名無しさん@英語勉強中:2015/12/09(水) 16:33:11.86 ID:HIjRPBEn.net
>>535
お薦めの歌教えて下さい

539 :名無しさん@英語勉強中:2015/12/09(水) 17:32:20.17 ID:1t8kLrg1.net
>>537
(1) But I never could fit in, it just wasn't me
  でも俺は、ぜんぜん溶け込めなかった。家の環境は、まったく俺向きじゃなかった。
(2) Searching for my dreams, in some far away place
  自分の夢を探しに(家を出たんだ)。どこか遠くにある夢をね。
(3) with this quest and my will to survive, to keep me alive
  夢を探し求める思いと、生き延びようという思いを抱いて。
(4) ※Hey hey show me you're a Cowboy, gotta do it now boy
  さあ、さあ、お前がカウボーイだってことを示してくれよ。
  今こそ、そうしてしてくれなきゃ、ダメだよ。

★文法事項については、具体的に何がわからないかを知らせてください。
 何もかも説明すると、何時間あっても足りません。

540 :537:2015/12/09(水) 18:30:18.10 ID:0ou3v9bB.net
>>539
親切にありがとうございます。
結局のところ私が口語文法に不慣れなのが悪いのですが、
(1)のit just wasn't me.の文法がわかりません。(文型も)
(3)のmy will to survive, to keep me aliveの間にコンマしかないのもよくわかりません。
(andなどがあれば文として通るのでしょうけど)
(4)のgotta do it now boyは
gottaがhave got toということは辞書に載っていたのですが、意味が今一分かりません。

541 :名無しさん@英語勉強中:2015/12/09(水) 18:32:48.68 ID:ZrGVD3kW.net
便乗して簡単な質問

I could never と
I never could は何か意味合い変わってくるんですか?

どっちの方が一般的なんでしょう

542 :名無しさん@英語勉強中:2015/12/09(水) 18:42:32.63 ID:HIjRPBEn.net
>>541
後者がより強意です

543 :三年英太郎 ◆3CZBjOt3.Y :2015/12/09(水) 18:46:35.17 ID:kPr1dx2k.net
簡単だと思うなら自分で調べたらええやん。自分で辞書引いた方が正確かつ勉強になる。

544 :名無しさん@英語勉強中:2015/12/09(水) 19:12:55.32 ID:1t8kLrg1.net
>>540
>>(1)のit just wasn't me.の文法がわかりません。(文型も)

  まず、It's me. という言い回しを覚える必要があります。この場合の it は、
  たとえば今回の場合は家族関係を意味します。家庭環境について It's me. と
  言うと、「うちの家庭環境は、俺向きじゃない」というような意味です。「俺の
  性格と親たちの性格がまったく違っていて、俺はそこに溶け込めない」というような
  意味合いです。

  洋服屋さんできれいな服を見て、It's me. というと、「この服は、俺向きだ。
  俺にピッタリだ。俺の好みに合う。俺に似合う」というような意味になります。
  このような It's me. It's you. It's us. It's them. It's him. などに
  ついては、以前に僕は長い長い解説をしたことがあります。興味があれば、
  下記のリンク先の解説を読んでください。でも覚悟してください。ものすごく長いです。

   http://yomogi.2ch.net/test/read.cgi/english/1389234352/158-161
   上記のリンク先の、158番から161まで。

(続く)

545 :名無しさん@英語勉強中:2015/12/09(水) 19:13:47.62 ID:1t8kLrg1.net
>>540 への回答の後半

>>(3)のmy will to survive, to keep me aliveの間にコンマしかないのもよくわかりません。
(andなどがあれば文として通るのでしょうけど)

  一つには、これは歌詞なので、口調を揃えるため、リズムに合わせて歌を作る
  必要があったため、その必要性に応じて and を省いたという理由があるはずです。
  次に、一般に、"AAA and BBB" という代わりに、AAA, BBB と言うふうに言ったり
  書いたりすることもあります。これは決して間違いではありません。ただ、初心者の
  うちからいきなりこれを真似しない方がいいと思います。初心者のうちは
  教科書通りに AAA and BBB と書くように心がけ、たまにネイティブが書いている
  AAA, BBB という書き方を見て、「ああ、こういう言い方をすることもあるんだな」 
  と思って流しておくことです。

>>(4)のgotta do it now boyは gottaがhave got toということは辞書に載っていたのですが、意味
が今一分かりません。

  I must [動詞] = I have to [動詞] = I have got to [動詞]
  = I've got to [動詞] = I got to [動詞] = I gotta [動詞]
  上記の最初の I must が最も正式で堅い言い回し。それからだんだんと
  口語的でくだけた言い方になっていき、I gotta は最高にくだけた言い方で、
  「〜しなきゃ」というような感じです。

546 :名無しさん@英語勉強中:2015/12/09(水) 19:16:37.61 ID:1t8kLrg1.net
>>544
書き間違えました。次の文章のうち、【  】の中が訂正後のフレーズです。

家庭環境について 【It's not me.】 と
  言うと、「うちの家庭環境は、俺向きじゃない」というような意味です。

547 :540:2015/12/09(水) 20:00:29.70 ID:Jx8NoyxH.net
>>546
丁寧なレスありがとうございます。リンク先の書き込み参考になりました。

548 :名無しさん@英語勉強中:2015/12/09(水) 20:01:19.76 ID:1t8kLrg1.net
>>540
AAA and BBB と書く代わりに AAA, BBB というふうにコンマを省くやり方について

実は、これについても僕は過去にいろいろと調べて、あちこちに書いてあることを
ここにすべて引用したことがあった。もしも面倒でなければ、次のリンク先を読んでほしい。

http://awabi.2ch.net/test/read.cgi/english/1398738149/992-996
  上記のリンク先の 992 番から 996 番まで

549 :540:2015/12/09(水) 21:27:53.75 ID:y8aWUH2v.net
>>548
丁寧なレスありがとうございます。

550 :名無しさん@英語勉強中:2015/12/09(水) 22:26:33.52 ID:BJIIseiU.net
>>535
恐ろしいまでに効率のいい勉強法の真逆を行っててワロタ

551 :名無しさん@英語勉強中:2015/12/09(水) 22:51:14.93 ID:ZumL4akL.net
mustとhave got toは違う意味なのに。

552 :名無しさん@英語勉強中:2015/12/09(水) 23:31:22.74 ID:4UWyhUJ6.net
You might be surprised by the locations of some of the world’s most radioactive places ?
and thus the number of people living in fear of the effects radiation could have on them and their children.

世界で最も放射能汚染されている場所にあなたは驚くことでしょう。
そしてその汚染地域に住んでいる多くの人々そして子供たちに、この放射能による影響がでる可能性があります。

という文なんですけど、

thus the number of people living in fear of the effects radiation could have on them and their childrenが、全く意味が分かりません。

553 :名無しさん@英語勉強中:2015/12/09(水) 23:41:37.78 ID:9GOyqNm/.net
てか、ここの人たちって無駄な勉強してるよなぁ

英語なんて中高生の英語がわかるレベルなら、あとは
ひたすら例文とか長文を50回反復して声に出して復唱するだけで
あとはその反復練習だけで英語脳ができるから普通にしゃべったり書いたりできるようになるで

例文とか長文とか1つを辞書ひいて訳して意味がわかったから次の例文とか長文ってやってるうちは
いくら勉強しても駄目、最低50回口に出して反復する

これだけでOK
劇的にかわるよ

554 :名無しさん@英語勉強中:2015/12/10(木) 00:05:31.77 ID:Fj9q5lbw.net
>>553
それやって、ただ読むだけがうまくなったやつ知ってるけど、何があなたと違うんだろう?

555 :名無しさん@英語勉強中:2015/12/10(木) 00:30:22.48 ID:1WfGQ2kR.net
must > have to = have got to > need >had better = should
こんなイメージだけど合ってる?

556 :名無しさん@英語勉強中:2015/12/10(木) 00:45:39.13 ID:m3zgmFxW.net
>>555
You have to see it from a different point of view.
The point is that you must quit 2ch and study harder.

'have (got) to' is used based on what people generally think of it, for example, when that is the way it ought to be.
'must' is used based on personal opinions, especially when the speaker is thinking that way.

557 :名無しさん@英語勉強中:2015/12/10(木) 02:26:50.63 ID:Jw+l1G9o.net
どのスレで聞こうかめっちゃ迷ったんだけど
ある日本かぶれ(?)のアメリカ人女性にとても気に入られて
(積極的なのか)一緒に住みたい、生活のことは全て任せろ、あなたの英語も私がなんとかする!、家は大丈夫
って渡米を促されてるんだけど
実際上手く生活できると思う?
まあ、自分自身、そこまで日本の生活が好きという感じでもなく外国で半生を過ごすのも悪く無いとは思ってるんだけど
何も見えない怖さってのがある、暗闇に向かってダッシュするような怖さ
ここの英語に精通しているみなさまは海外の生活(マスオさん)はどう思いますか?
あ、英語のレベルは中学卒業レベルですwスレチかもしれないけどw

558 :名無しさん@英語勉強中:2015/12/10(木) 03:46:03.02 ID:D69X3rvd.net
>>557
ヒント
多額の生命保険

559 :名無しさん@英語勉強中:2015/12/10(木) 05:00:01.20 ID:xaR51Mz9.net
動画でリスニング鍛えてるんだけど
youtubeのreutersはなんで更新されないので
使えなくなってしまいました。
知ってるひといたらおしえていただきたいのですが

560 :名無しさん@英語勉強中:2015/12/10(木) 05:48:05.27 ID:IJbNSSwU.net
She was indeed as beautiful as everyone said she was.

この文がbe動詞で終わるのがよーわからんのです。

疑問詞がある場合にbe動詞で終わるパターンは見たことがあるんだけれど
その場合は疑問詞以下の語順に倒置がおこらないから
下記みたいな感じ

who is this? → who this is?

今回のは疑問詞もないし疑問文でもない
どういうこと?

561 :名無しさん@英語勉強中:2015/12/10(木) 06:48:38.26 ID:q+xk7s6s.net
>>540
>>(3)のmy will to survive, to keep me aliveの間にコンマしかないのもよくわかりません。

昨日は、いろいろと長文で回答しましたが、この件については、間違った回答をしてしまいました。
ここでは、to survive と to keep me alive は、ほとんど同じ意味です。
この二つは同格の関係にあるのです。だからこの二つは and で結んではいけない
のです。今回のようにコンマにするか、あるいはコロン( :)か、あるいは "M dash"
(すなわち — という長い棒)をつけるべきところだと思います。

それから、must = have (got) to と書いてしまいました。まったく同じ意味であるかの
ように書いてしまいましたが、それは他の人の言うように、不正確でしたね。すみません。
僕が言いたかったのは、「義務(しないといけない)」という点ではだいたい同じ意味だ
という程度のことでした。

562 :名無しさん@英語勉強中:2015/12/10(木) 07:05:23.42 ID:q+xk7s6s.net
>>552
"M dash"(横に長い棒)が「?」の符号に文字化けしてますよ。質問文がきちんと
表示されているか、きちんと点検してくださいね。

You might be surprised by the locations of some of the world’s most radioactive places
  世界で最高に放射能の強い地域のうちの一部がどこにあるかということと、

— and thus the number of people living in fear of the effects
[which] radiation could have on them and their children.
  それゆえに自分やその子供に対して放射線が及ぼしうる影響に脅えながら
  生活している人々の数を見て、みなさんは驚くかもしれません。

原文の出典:
   http://brainz.org/ten-most-radioactive-places-earth/

563 :名無しさん@英語勉強中:2015/12/10(木) 08:49:11.06 ID:xaR51Mz9.net
答えまだ?

564 :名無しさん@英語勉強中:2015/12/10(木) 09:26:38.93 ID:q+xk7s6s.net
>>560
>>She was indeed as beautiful as everyone said she was.

質問の趣旨がよくわからないんだけど、次のような例文を見て、それと同じようなものだと
納得することはできないだろうか?

(1) I am as tall as Bill (Bill is).
(2) Mary is as old as Jane (is).

上記の (1), (2) では、通常は最後の is をつけないけど、正式にはくっつく。
それと同じようなことが、今回のあなたの質問にあった文にも起きているのではないか?

565 :名無しさん@英語勉強中:2015/12/10(木) 12:20:57.90 ID:u9e8Hc70.net
We should so act as to have nothing to regret.
という例文を見かけたのですが
so as to構文のsoの後に動詞が来る形というのを初めてみました
これは何か特殊な用法なのでしょうか

566 :名無しさん@英語勉強中:2015/12/10(木) 12:28:36.81 ID:idprliYj.net
The way such crowded places respond to climate change will largely determine whether global warming can be slowed

これのsuch〜changeって関係副詞ですよね?

567 :名無しさん@英語勉強中:2015/12/10(木) 12:38:43.45 ID:be7uQv2P.net
We recognize our responsibility <as a big >
to serve <as an enviromental leader>

この文のas asってただの〜としてって事ですか?そしてto serveはour responsibilityに
かかっていますか?

568 :名無しさん@英語勉強中:2015/12/10(木) 12:56:09.83 ID:JI8V25dg.net
「You ain't no Muslim, bruv(あんたなんかムスリムじゃないぜ)」

ニュースで話題のこの文は逆の意味では?
あなたがムスリムじゃ無いなんて言わせない
あなたはムスリムだ!

569 :名無しさん@英語勉強中:2015/12/10(木) 13:05:49.01 ID:0hHvHyul.net
>>564
ちがくない?

everyone said she was (beautiful).
みんなは彼女が美しいと言った

これが正解だけど (beautiful) は文中手前の as beautiful as で明示されてるから
繰り返しをさけるために省略されたってことだと思うけど

570 :名無しさん@英語勉強中:2015/12/10(木) 13:07:36.87 ID:oFoJ9gL+.net
英語で文章を書こうとしてる者です。

たとえば、She drives me crazy.っていう文が有ったとして、その意味で他の言い回しが
いくつも掲載されてるようなサイト、本、辞書など無いでしょうか?
synonymとか単語だけならあるのですが、
フレーズとか文とかでそういうの探してるんです。
アメリカのAmazonとかで探せば良いでしょうか?お願いします。

571 :名無しさん@英語勉強中:2015/12/10(木) 13:16:18.59 ID:ntfPkXOv.net
>>557
いのちだいじに

572 :名無しさん@英語勉強中:2015/12/10(木) 13:32:58.27 ID:q+xk7s6s.net
>>565
特殊ではありません。そういうのは、他にもときどき見かけます。今、ネットで見つけた
ものを列挙してみます。

(1) he should ●so behave as to● oblige me to leave him
(2) It is the duty of a corporation which constructs a bridge
across a navigable stream to ●so build it as to● interfere as little as possible with navigation,
(3) we should ●so organize it as to● secure efficient, independent
and enlightened administrators of the law,

他にも探してみれば、短時間でいくらでも見つかると思いますよ。

573 :えワ:2015/12/10(木) 13:38:21.41 ID:LSCM/TPa.net
IDを変えて質問する必要はないだろう。

>>565
>We should so act as to have nothing to regret.
We should act nothing to regret. で足りてる。
so 〜 as to have は「不要な単語」
自分で使う気が無ければ、意味が取れたら、それ以上は考えない方がいい。

>>566
way にかかる形容詞句。
前文、後文がないと、way が単数の理由が分からない。

>>567
「責任は大きい」の日本語のをそのまま英訳したんだろうね。
日本人の英文、英語として考える意味はないと思う。

>>568
前後の文脈が無いと判断出来ない質問だね。
むしろ、bruv のお話をした方がいいんじゃないのかな。


>>560
Eerybody knows she was beautiful. で足りてる。
文法用に作った文章だろうから、自分で使う気が無ければ、意味が取れればそれでいい。
文法の話をするには、英語の知識がまだ、足りない。

574 :名無しさん@英語勉強中:2015/12/10(木) 13:39:35.56 ID:q+xk7s6s.net
>>568
You ain't no Muslim, bruv
  = You aren't a Muslim, brother!

これは、bruv という単語から判断して、たぶん Cockney 方言だと思うけど、
違うかな?それはともかく、"ain't no" = isn't a (or any) という意味。
二重否定は一重否定と同じ。それどころか、5重否定でも1重否定と同じ。
例えば、次のような感じ。もともとイギリスの一部の地域にある方言だった
ものが、アメリカにわたり、一部の白人や黒人のあいだで定着したもの。
南部の黒人は、特にこのようなしゃべり方をしているようだ。

I ain't never done nothing wrong to nobody nowhere, nohow.
= I haven't ever done anything wrong to anybody anywhere, anyhow.

575 :名無しさん@英語勉強中:2015/12/10(木) 14:54:03.47 ID:pR7FsYaq.net
>>574
568です

>>"ain't no" = isn't a (or any) という意味。
二重否定は一重否定と同じ。それどころか、5重否定でも1重否定と同じ。

有難うございます
スッキリしました

576 :名無しさん@英語勉強中:2015/12/10(木) 15:13:12.88 ID:kN+aZhl0.net
Misty kind of dayとA Misty dayとの間にはどのような差異があるのでしょうか?
あと、diamond- the hardest kind of stone
などの場合もkind ofがなくても意味が通ると思うのですが、kind ofはどのような意味なのでしょうか?

577 :えワ:2015/12/10(木) 15:18:39.63 ID:LSCM/TPa.net
>>575
状況を考えると、
You ain't, no Muslim bruv ! → You are not right. You are NO Muslim !
「違うね、イスラムを騙(かた)るな、クズ!」と取るべきだろう。
文法的には、二つの否定文、二重否定とかの解釈は間違いだと思う。

ロンドンの北東にある地下鉄Leytonstone駅で一人の男性が通行人を突然刺すという事件が発生した。
一人に重傷を負わせ、さらに二人を傷つけた男性は多数の警察官に取り押さえられた。
男は「シリアのためだ」と叫んだ。
そのとき、周囲で見ていた中から、“You ain’t no Muslim, bruv !” の声がした。

記事と質問のレベルを考えてほしいけど、無理だろうね、君は馬鹿だから。

578 :えワ:2015/12/10(木) 17:02:10.28 ID:LSCM/TPa.net
>>576
婉曲的で意味がちょっと弱くなる。
「〜みたいな」「いわゆる」と訳しておくと、座りがいいと思う。
自分で使う時は、「不要な単語」にならないように、しっかり意味を持たせる意識がいる。

↓こんなイメージ。

あいつは馬鹿だ。
あいつみたいな奴は馬鹿だ。
あいつは、いわゆる「馬鹿」だ。

ダイヤモンドは固い。
ダイヤモンドは硬度規格では、最も固い。

これが分かりやすいかな。
This is necessary. これは必要な物。
This is a kind of "necessary." これはいわゆる「必要な物」 ← 他にも必要な物がある事を示唆している。
This is co-called the "necessary." これはいわゆる「必要な物」

579 :名無しさん@英語勉強中:2015/12/10(木) 18:00:55.92 ID:Wz3SOodk.net
ある書類をもらいたいとき、下記のような書き方は可でしょうか?Let me know…という文章はよく見ますが、それ以外の使い方ってあるのかなと思いまして。

Please let < a document> issued to us …

580 :えワ:2015/12/10(木) 18:14:35.81 ID:LSCM/TPa.net
>>579
定型文があるだろう。
「馬鹿な考え、休むに似たり」だね。
考える方向を考えた方がいいね。

581 :名無しさん@英語勉強中:2015/12/10(木) 18:32:34.91 ID:q+xk7s6s.net
>>579
下記のものは、すべてネットで見つけたもの。

(1) ... please ●send us● a Form W-9 with your correct Tax ID number.
(2) Please ●furnish us with● a document which spells out
the various review procedures which are....
(3) please ●issue me● a certificate of work in your firm

582 :三年英太郎 ◆3CZBjOt3.Y :2015/12/10(木) 18:32:51.47 ID:T1milFzQ.net
>>579
それ以外の使い方を知りたいなら、辞書で文例みたら?
その文章が文法的に間違ってることがついでにわかる。

583 :えワ:2015/12/10(木) 18:38:48.57 ID:LSCM/TPa.net
>>581
「何の書類」と聞かないといけないんだけどね。
まぁ、自演とバレてるよ。

584 :名無しさん@英語勉強中:2015/12/10(木) 18:51:23.89 ID:cGlsh1Ot.net
1つ質問させて頂きます。
The adhesive strength is approximately 30% lower after sample treatments with arc-discharge in relation to the sample as delivered.
30% lowerはどのように使われているのでしょうか。
・接着強度の値は、after以下より

585 :名無しさん@英語勉強中:2015/12/10(木) 18:53:48.99 ID:cGlsh1Ot.net
失礼しました。文が途切れていたので再送します。

The adhesive strength is approximately 30% lower after sample treatments with arc-discharge in relation to the sample as delivered.
30% lowerはどのように使われているのでしょうか。

・接着強度の値は、after以下より「約30%低い」。
・接着強度の値は、after以下より低くて「約30%である」。

宜しくお願い致します。

586 :777 ◆TFWBMdHdF7zL :2015/12/10(木) 18:58:37.80 ID:ANl+aXzl.net
>>572
例えば so behave as to で Google Books を検索すると 19 世紀以前のものがほとんどなんだが。
特殊でないなら現代でも多くの用例が見つかっていいはず。

587 :えワ:2015/12/10(木) 19:01:28.11 ID:LSCM/TPa.net
>>585
arc-discharge の説明がないと誰も答えられないね、自演以外。
まぁ、チンコピアスが答えるんだろう。

>>586
本物の777氏なら、その答えまで書くだろうね。

588 :名無しさん@英語勉強中:2015/12/10(木) 19:14:15.43 ID:cGlsh1Ot.net
>>587
arc-dischargeはアーク放電のことらしいです。
サンプルをアーク放電で処理した後
・処理前と処理後の接着強度の「差」が30%なのか
・処理後の方が接着強度が下がって「値」が30%であるのか
が知りたいです。

589 :えワ:2015/12/10(木) 19:23:46.96 ID:LSCM/TPa.net
>>588
30% lower
意味が分からないのかな。

「信頼性試験」でググって知識を増やした方がいいだろうね。
英語ではなく、日本語の問題だよ、偽777君。

590 :名無しさん@英語勉強中:2015/12/10(木) 19:32:39.01 ID:cU8FiEDB.net
The adhesive strength is approximately 30% lower after sample treatments with arc-discharge in relation to the sample as delivered.


粘着力は送ったようなサンプルと比べ、アーク放電試料処理後、およそ30%弱くなる。

591 :名無しさん@英語勉強中:2015/12/10(木) 19:35:40.53 ID:cGlsh1Ot.net
>>590
ありがとうございます

592 :エワ:2015/12/10(木) 19:44:56.42 ID:LSCM/TPa.net
>>591
紫外線の影響=直射日光だからね。
「直射日光下では接着強度が低下します。」という意味だね。
まぁ、英語ではなく、日本語の話だ、偽777君。

593 :名無しさん@英語勉強中:2015/12/10(木) 19:55:31.81 ID:rNDqeAJj.net
>>590
in relation to は 〜に関して じゃね

594 :名無しさん@英語勉強中:2015/12/10(木) 19:58:30.12 ID:PnnUpC32.net
初歩的な質問ですいません。

海外の女性音楽グループに来日してもらいたいという意味で、
「私たちは日本であなたたちに会いたい」というメールを送りたいのですが

We'd like to meet you in Japan.

でよいのでしょうか?

595 :えワ:2015/12/10(木) 20:09:58.78 ID:LSCM/TPa.net
>>594
音楽グループなら、「会いたい」じゃなく「演奏が聞きたい」と書くべきだね。

596 :名無しさん@英語勉強中:2015/12/10(木) 20:12:17.11 ID:0b/wtdyl.net
>>581
定型文があることは承知なのですが(こんなビジネスで頻出のシチュエーションはググればいくらでも出てくるでしょうね)、letをこういう風に使えないか、もし使えないのならどこがどうおかしいのかが知りたいのです。

597 :名無しさん@英語勉強中:2015/12/10(木) 20:27:14.97 ID:yVdwuK7z.net
たとえば、ある大会の話題をだしてる場合は
I'll be the champion!ですよね。

そこで、
I'll be a champion! と書いてた場合、どういう意味が読み取れますでしょうか?

おそらく冠詞の使い方を間違えてるんだと思いますが、the以外ありえないですよね。

598 :えワ:2015/12/10(木) 20:30:47.72 ID:LSCM/TPa.net
>>596
let は強い単語でね、質問するレベルなら考えない方がいい。
「有無を言わせない強さ」なんで、私は使わない。
let は選択肢を許さないんだよ。

>>597
第一会大会なら a でもいいかも。
で、一回しかなければね。

599 :名無しさん@英語勉強中:2015/12/10(木) 20:40:50.64 ID:q+xk7s6s.net
>>596
let の使い方なんて、他の人が言った通り、数種類の辞書を見ればいくらでも
わかるし、辞書に載っていないことは、コーパスを使っていろんなものを入れていって、
たくさんの用例が見つかればそれは正しいのだろうし、ほとんど用例が見つからなければ
ダメなんでしょう。ともかく、ネイティブらしき人が使っていることが確実だと思われる
用例が見つからないなら、使わないことです。なお、小さな辞書なんか見たってだめです。
少なくともウィズダム英和とジーニアス英和だけは見るべきですね。もしそれが面倒なら、
最初からそんな偉そうな態度で答えだけを求めないようにすることです。

600 :えワ:2015/12/10(木) 21:01:02.71 ID:LSCM/TPa.net
>>599
日本語は苦手?

601 :名無しさん@英語勉強中:2015/12/10(木) 21:30:07.31 ID:RwzyBGpK.net
not only but also を使わず

I like cat.But I like dog as well.

会話で使う時にこの表現は問題ないでしょうか?
butがいらない気もしますが微妙なニュアンスっぽいので分かりません

602 :えワ:2015/12/10(木) 21:33:46.98 ID:LSCM/TPa.net
>>601
I like dog and cat. で足りるね。
英語が出来ないのに、スレを荒らすのは、大変だね。

603 :名無しさん@英語勉強中:2015/12/10(木) 21:37:24.82 ID:wfbW/8W/.net
There has come to be something almost impious in denying that all societies and all human problems this side of mortality can be radically made over and suffering brought to an end.
And by definition, in a progress narrative, the good eventually triumphs and the bad is defeated.
In this universe, perhaps more emotional (not to say narcissistic) than it is moral in the proper sense,
it is almost an assertion of one’s membership in the party of the good not to treat sceptically the claims that the world is on the cusp of being made anew that are routinely made by development and aid officials from major Western governments.

all societies and all human problems this side of mortality can be radically made over and suffering brought to an end
この文の構造がわかりません 解説お願いします
あとclams thatの2つのthatは一番目が同格で2番目が関係代名詞であってるでしょうか?

604 :えワ:2015/12/10(木) 21:50:38.24 ID:LSCM/TPa.net
>>603
チンコピアス、いらっしゃい。
端末がね。
もう少し、違う踊りを見せてよ。

605 :名無しさん@英語勉強中:2015/12/10(木) 22:24:02.77 ID:mqrXGDCO.net
>>603
この世の全ての社会や全ての人間的問題を
根本から刷新し、苦しみに終止符を打つこ
との可能性を否定することは、何時からか、
どことなく不信心な響きを帯びるようにな
った。そして、当然ながら、進歩の物語に
おいては、正義は最終的に勝利し、悪は負
ける。この夢世界は、厳密に言うと、道徳
的と言うよりは感情的(ナルシスト的であ
ることは言うまでもなく)なものであるが、
そこでは、世の中が新しく作り変えられる
寸前のところまで来ているという、西側の
主要諸政府の発展や援助に関係する役人が
決まって言う主張に、疑いの眼差しを向け
ないことが、自分が正義の味方の一員であ
ることの表明とほぼ等しい行為なのである。

this side of mortality は all societies and all human problems に対する後置修飾語句。
suffering (can be) brought to an end は空所化変形。
一つ目の that は claims に対する同格。
二つ目の that は claims を先行詞とする関係代名詞。

606 :名無しさん@英語勉強中:2015/12/10(木) 23:23:01.07 ID:q4QJ2/ns.net
食料が欲しければ奪い、女が欲しければ犯すという欲望を剥き出しにした振る舞い

というのを英語で表現するとどうなるのでしょうか?

607 :名無しさん@英語勉強中:2015/12/10(木) 23:56:53.03 ID:9lpOf+TM.net
英語に翻訳して欲しい文があるんだけどいいかな?

608 :名無しさん@英語勉強中:2015/12/11(金) 00:00:33.11 ID:KFf4sZNC.net
英語に役立つのはドイツ語とフランス語じゃどっちですかね?
どちらかを必ず学ばなくてはならないんです

609 :名無しさん@英語勉強中:2015/12/11(金) 00:43:32.89 ID:YzK20l5k.net
Was it fine or cloudy?
に対する返答の仕方は晴れていた場合、
Yes, it was fine.とIt was fine.どちらが正しいのですか?

610 :名無しさん@英語勉強中:2015/12/11(金) 06:34:29.69 ID:5eafb7eZ.net
>>606
食料が欲しければ奪い、女が欲しければ犯すという欲望を剥き出しにした振る舞い

もしそれを直訳に近く訳すとしたら
(1) a behavior that is a naked expression of one's desire to
  steal food when hungry and to violate women when aroused

もしそれを一つの文として表現しなおすとしたら
(2) They (= The criminals) acted in a naked manner on their desire
  to steal food when hungry and to violate women when aroused.
(3) Their conduct was a naked expression of their desire
  to steal food when hungry and to violate women when aroused.

611 :名無しさん@英語勉強中:2015/12/11(金) 06:39:06.53 ID:5eafb7eZ.net
>>601
会話したいなら、まずは基礎英語をマスターした後にすることですね。
cat, dog のままだったら、「猫や犬の肉を食べるのが好き」という話になってしまう。

  (1) I like cats. I like dogs as well.
  (2) I like both cats and dogs.
  (3) I like cats—and dogs too.

612 :名無しさん@英語勉強中:2015/12/11(金) 06:51:02.86 ID:5eafb7eZ.net
>>608
数十年にわたって英語を勉強するつもりであり、第二外国語は(あとになって忘れても
いいから、少なくとも一時的には)日常会話がある程度はできる程度にまで習得する
つもりなのであれば、長い目で見ればドイツ語もフランス語も両方とも、同じように
役立ちます。

ただし、差しあたって大学での単位を取るだけで、第二外国語をあまり真剣に勉強する
気がないのであれば、ドイツ語を取れば楽です。ドイツ語は、学習の初期(具体的には
最初の少なくとも 1,000時間のあいだ)は、英語を多少とも知っていれば実に楽に
習得できます。簡単な単語や簡単な文法事項が、すべて英語に似ているからです。

しかし、第二外国語を少なくとも毎日1時間、そして何年もやり続ける覚悟があるのなら、
フランス語を勉強した方が、ゆくゆくは得です。英語の抽象的な単語が、フランス語に
似ているどころか、まったく同じ部分も多いからです。さらには、政治経済、ビジネスや
学問芸術でフランス語が使われることが多いので、あとで非常に助かります。

ただ、英語を本気でやり続けるつもりなのであれば、どちらか一方で終わらせず、
ゆくゆくは両方とも、少なくとも1年ずつ勉強しておくことを奨めます。

613 :名無しさん@英語勉強中:2015/12/11(金) 11:01:16.40 ID:OJUshYve.net
>>605
ありがとう

614 :名無しさん@英語勉強中:2015/12/11(金) 11:29:54.67 ID:+TqGV0gF.net
初歩的な質問で申し訳ないのですが、
have a ballで「楽しむ」という意味になりますが、
「二度楽しむ」はhave a ball twiceで、have two ballsとは言わないのでしょうか?

他の例で見ると、例えば「もう一度挑戦する」といった場合に、
try againとanother tryの両方害受けられるのでhave a ballなどの句動詞の場合どうなるのか
わからんおですが。。。

have two ballsだと下品な意味にもなりますね。

615 :名無しさん@英語勉強中:2015/12/11(金) 11:43:24.60 ID:PHR8oocH.net
句動詞てw

616 :名無しさん@英語勉強中:2015/12/11(金) 12:07:45.04 ID:5eafb7eZ.net
>>614
そうですね。have two balls とは、たぶん言わないと思います。「二度楽しむ」は、次の
ように言えばいいと思います。

(1) have a ball twice
(2) have fun twice
(3) enjoy it twice
(4) enjoy myself twice

617 :614:2015/12/11(金) 12:48:53.61 ID:HGdIHnjv.net
>>615
確かに句動詞ではないですね。イディオムといえばよかったか・・・
>>616
レスありがとうございます。参考になりました

618 :名無しさん@英語勉強中:2015/12/11(金) 15:36:20.33 ID:3b5b6yqi.net
A general strike,it said, had been called all over the United States.

この英文って元は
it said that a general strike had been called all over the United States

で良いですか?

619 :名無しさん@英語勉強中:2015/12/11(金) 15:37:42.60 ID:mRclQTcp.net
2chを読み込むと、ソフトウェアが自動でレスするエワというソフトはもう市販されていますか

620 :名無しさん@英語勉強中:2015/12/11(金) 17:11:09.35 ID:4jLPi+cZ.net
これはどう訳すんでしょうか
> I am english is cut no but
> you of think is good and think.

621 :名無しさん@英語勉強中:2015/12/11(金) 18:28:50.48 ID:EVuN007O.net
 
He could assist me with a batch.

これってHeは作業を手伝ってくれた?くれなかった?どっちなんでしょうか

622 :名無しさん@英語勉強中:2015/12/11(金) 18:50:43.70 ID:tr+eq4qS.net
Ms. Lu's assistant will mail a copy of the annual report to ___ who cannot attend the meeting.

(A) them
(B) this
(C) those
(D) then

という問題で、答えは(C)なのですが、さらっと「先行詞となれるのは(C)のみ」と書かれています。
(A)を選んでしまったのですが、(A)が先行詞になれない理由を教えてください。

623 :名無しさん@英語勉強中:2015/12/11(金) 18:54:35.57 ID:5eafb7eZ.net
>>621
その一文だけでは、二つのまったく違った意味になりうるので、よかったら、
その前の一文、できれば二つの文をここに引用してください。

624 :名無しさん@英語勉強中:2015/12/11(金) 18:55:46.33 ID:2ovb69wn.net
英語の冠詞の選択は、話者の主観で比較的自由に選択可能なんでしょうか。

もちろん相手に理解してもらうのが言語の目的なので自由度は
ある程度制限されるでしょうが、
少なくも聞き手や読み手も話し手や書き手の「期待」で選ばれている
ということを了解しているものなんでしょうか。

言語学でいうモダリティのようなものに含まれますか?

625 :名無しさん@英語勉強中:2015/12/11(金) 19:37:58.07 ID:5eafb7eZ.net
>>624
僕の偏見かもしれませんが、具体的な英語の例文を提示しながら行う具体的な
論議にしか意味がないと思います。もしあなたのように大上段に立った抽象的な議論が
できる人がいたとしたら、英検1級程度ではとうてい足りず、ネイティブそっくりな
英語力と、さらには言語学への深い造詣が必要だと思います。
そこまでの英語力のない人が言語学だけをかじって抽象的な議論をする人が
たくさんいますが、小学生が社会改革を論じるのと同じく馬鹿げていると思います。

626 :名無しさん@英語勉強中:2015/12/11(金) 19:58:05.96 ID:/2A/OWKx.net
What are the respective rates of cruel/scary persons and kind/nice persons among staffs of your company?

という文ですが、手直しするべきところはありますか?

627 :名無しさん@英語勉強中:2015/12/11(金) 20:21:15.93 ID:HGmfC1qn.net
ベテランの教師でも良い子に目がいってしまう。
Even veteran teachers pay attention to the good kids.

この場合、
Even a veteran teacher pays attention to a good kids.
だと意味がかわりますでしょうか?
ベテランの教師でも良い子に目がいってしまうものだ。のように一般化を宣言した場合、ともに不定冠詞で対応できるのでしょうか?

また、
教師の言うことが常に正しいとは限らない。
What a teacher says is not necessarily correct.

この場合は a teacherのみで what teachers say is 〜

のようにできないのでしょうか?

628 :名無しさん@英語勉強中:2015/12/11(金) 20:23:50.40 ID:5eafb7eZ.net
>>626
>>What are the respective rates of cruel/scary persons
and kind/nice persons among staffs of your company?

--> What are the percentage of cruel, scary persons
and that of kind, nice persons in the workforce of your company?

629 :名無しさん@英語勉強中:2015/12/11(金) 20:24:55.27 ID:HGmfC1qn.net
Even a veteran teacher pays attention to a good kid. ですね。訂正します

630 :えワ:2015/12/11(金) 20:32:38.75 ID:Y/RFclWd.net
>>622
文法用に作った英文だから考え過ぎずに、
「report to them who 〜の形は、 them who が、those who の形になる」でいいよ。

>>624
冠詞は文意を支配しないから、好きなようにしていい。
「冠詞は特殊な用例を持った形容詞」なんで、形容詞としての解釈になる。
基本のルールは守るとして、それでも、何を付けて良いか迷う時は、何を付けてもいい。

>>626
答えようのない質問になってるから、手直しする以前に、ダメだね。
ISOがあるんだから、それを参考にした方がいいだろう。

社員の評価の基準はあるか。
ある場合、「運用中の規範を基準を提示して下さい」が形だろうね。
Are there any documeted standard to estimate staffs or operators ?
If you have, please submit the documents

>>629
>a good kids. 変わらない、冠詞は文意を支配しない。.
>teachers は日本語になっていない。

631 :名無しさん@英語勉強中:2015/12/11(金) 20:35:43.81 ID:HGmfC1qn.net
>>630
ありがとうございます。

what teachers say is not necessarily correct.

これだとなぜ、文にならないのでしょうか?

632 :名無しさん@英語勉強中:2015/12/11(金) 20:52:31.80 ID:2ovb69wn.net
>>625
文法についての理解はそういう意味ではみんな抽象的ですよ。
具体例だけにいちいち個別に依拠するなら文法なんて不要ですしね。
この例はこうだけど、別の例ではちがうの?という話にぜったいなるでしょ。
だから文法のような抽象的な概念理解が必要になるんじゃないですか?

>>630
どうもありがとう。
根本的には話者の期待ですよね?
相手も了解しているだろう、察してくれるだろう、という。

633 :名無しさん@英語勉強中:2015/12/11(金) 20:59:02.59 ID:5eafb7eZ.net
>>632
そう思うのなら、僕には何も言うことはありません。小学生が社会改革を論じているとき、
大人は黙って見ているしかない。本人は社会を理解しているつもりでいるんですから。
僕が言っているのは、社会に出て働いたこともない人が社会の構造を抽象的に
云々するのは無意味だと言っているだけです。同じように、ろくに英語ができない人が
抽象的に文法を論じるのも無意味です。ただし、あなたが英検1級を20年前に
突破し、そのあとずっと言語学か翻訳か通訳のプロとして働いてきたのなら話は別です。

634 :名無しさん@英語勉強中:2015/12/11(金) 21:02:55.22 ID:L7wcRQgl.net
>>624
言葉なのだから一定の決まりがあるはず。625に同意だ。
英語を日本語で議論しても日常会話もできないのでは意味がないよ

635 :三年英太郎 ◆3CZBjOt3.Y :2015/12/11(金) 21:38:42.44 ID:4lImTWM+.net
>>627
老婆心ながら助言しておくと、その類の質問は頻出してるので、スレ住人も飽きてるだろう。
しかし超重要部分なので、ここで中途半端な回答(つかキチガイの誤回答)を信じるより、
文法書の該当部分を一読した方がよろしかろう。

636 :名無しさん@英語勉強中:2015/12/11(金) 21:41:58.64 ID:HGmfC1qn.net
>>635

>>627は自分が文法書を読んだ上での結論です。数冊参照してるので、もしよろしければ、確認できそうな文法書などありますでしょうか?

よろしければ、これ>>631 だけでも教えてほしいのですが

637 :えワ:2015/12/11(金) 21:54:00.59 ID:Y/RFclWd.net
>>634
IDを変えてもね、まぁ、自演は止めようね。

>>636
こんな意味になるね。
人殺しの宮本賢治にだまされてる日教組の教師の言う事は、信用出来ない。

638 :名無しさん@英語勉強中:2015/12/11(金) 22:16:01.74 ID:L7wcRQgl.net
自演の見本は、前にエワ本人がやった10分ごとにレスしたのがわかりやすい。

今回のは書き込みのタイミングが重なっただけだよ

639 :えワ:2015/12/11(金) 22:24:31.99 ID:Y/RFclWd.net
>>638
頭の中身が同じだから、他人には見えないよ。

640 :三年英太郎 ◆3CZBjOt3.Y :2015/12/11(金) 22:48:29.24 ID:4lImTWM+.net
>>636
ちなみにどの文法書?

> Even veteran teachers pay attention to the good kids.

the+複数名詞は特定のグループを指す。
ので a good kid には置き換えられない。
なお、不定冠詞の総称表現は例外が多くて特に難しいので、
とりあえず困ったら無冠詞+複数名詞を使うことをオススメする。

What a teacher says is not necessarily correct.
What teachers say is not necessarily correct.

どっちもOK.

641 :名無しさん@英語勉強中:2015/12/11(金) 22:57:14.21 ID:PHR8oocH.net
What Eitaro says is not correct.

642 :名無しさん@英語勉強中:2015/12/11(金) 22:59:48.78 ID:aUnGqAJ+.net
>>630
>冠詞は文意を支配しないから、・・・・・・

支配とまでは言わないが、文意を限定するんじゃない? 
冠詞によって意味が変わるから、好きに使うわけにはいかない。

643 :名無しさん@英語勉強中:2015/12/11(金) 23:07:22.60 ID:L7wcRQgl.net
>>626
persons→people
staffs→staff members

644 :三年英太郎 ◆3CZBjOt3.Y :2015/12/11(金) 23:25:16.72 ID:4lImTWM+.net
>>640の前半は、意味が変わるので the kids → a kid には代替できない、という意味ね。
念のため。

645 :名無しさん@英語勉強中:2015/12/11(金) 23:35:25.60 ID:HGmfC1qn.net
>>644
What a teacher says is not necessarily correct.

で a teacherが あたかも、教師というものは、 という意味になるのはなぜでしょうか?

What teachers say is not necessarily correct.

こちらは文字通り、教師というものは、という意味になりますが、What a teacher says
とのニュアンスの違いなどはありますでしょうか?

646 :三年英太郎 ◆3CZBjOt3.Y :2015/12/11(金) 23:49:15.40 ID:4lImTWM+.net
だから、文法書ひきなさいってばよ(⌒−⌒;)
そんなこと聞いてる時点て、本当は調べてないでしょ(⌒−⌒;)

持ってないなら、今日のところは「総称 冠詞」とかで検索して、
明日買って来ましょう。

647 :名無しさん@英語勉強中:2015/12/12(土) 00:11:20.84 ID:9fDMDAkW.net
スレタイとテンプレ読んでこい
調べる方が楽で正確なのにわざわざここで聞くのはここに住み着いてる奴への温情だろ

648 :名無しさん@英語勉強中:2015/12/12(土) 00:15:48.19 ID:EyMBC7Vf.net
英語話せないやつに限って
なんでこんなに文法の質問ばっかなんだろ

649 :名無しさん@英語勉強中:2015/12/12(土) 00:16:28.79 ID:q//iCzD4.net
文法が分かってないから話せないんだよ

650 :名無しさん@英語勉強中:2015/12/12(土) 00:59:38.52 ID:GgguyG8W.net
英語に文法は必要ねーよ

651 :名無しさん@英語勉強中:2015/12/12(土) 01:01:40.49 ID:mxomBQgX.net
え?w

652 :名無しさん@英語勉強中:2015/12/12(土) 01:33:44.95 ID:XNucQtnh.net
>>623
ゲームの行動結果を示す短い文章で、データ上では

10001 彼は作業を手伝ってくれた。
10002 >>621
10003 彼は作業をただ見ていた...

こうなってるんで
>>621の文は最終的にHeが手伝ってくれたのかくれなかったのかマジで分からない状態です
この一文だけではどっちか分からないとなるとゲームで検証するしかないか・・・ありがとうございました

653 :名無しさん@英語勉強中:2015/12/12(土) 01:35:09.50 ID:XNucQtnh.net
Heはその作業を手伝うスキルはあるんだけど
meのことが嫌いなので手伝ってくれなかった
っていうのもシステム的にありうるので
分からん・・・

654 :名無しさん@英語勉強中:2015/12/12(土) 02:48:05.64 ID:nge4wWo1.net
>>622
me whoとかを見たことがないのと同じ理由

655 :名無しさん@英語勉強中:2015/12/12(土) 02:58:18.57 ID:q//iCzD4.net
つーかそれ中学で関係代名詞習う時に習うはずだろw>>622

656 :名無しさん@英語勉強中:2015/12/12(土) 03:43:42.87 ID:EyMBC7Vf.net
>>649
アメリカ人の大半は文法は習わないがペラペラだよ

657 :名無しさん@英語勉強中:2015/12/12(土) 04:04:44.62 ID:mxomBQgX.net
出たーw何もわかってない奴の常套句www

658 :えワ:2015/12/12(土) 04:08:20.73 ID:MqYG+ov/.net
>>656
アメリカ人にとって、英語は「母国語」だからね。
英語は日本人にとって、「外国語」なんで、同じ扱いは出来ないんだよ。

イェスペルセンに始まる「どうやって外国語を学ぶのか」は、今でも言語学の最重要課題だからね。
ただ、方法論より、個人の適正の方が大きくて、「脳生理学(言語野)が不可欠←いまここ」という所。

659 :名無しさん@英語勉強中:2015/12/12(土) 06:11:31.62 ID:EyMBC7Vf.net
>>658
んじゃ文法学んでもなんであんた英会話もできない低レベルなんだ?

660 :えワ:2015/12/12(土) 06:23:35.79 ID:MqYG+ov/.net
>>659
「読む英語」と「書く英語」と「話す英語」は別物なんだよ。
それぞれ、求められる物が違う。

いつまでも、「ガキの遊び」が通用すると思うな。

661 :名無しさん@英語勉強中:2015/12/12(土) 08:41:07.11 ID:M64tIOIt.net
>>649
>文法が分かってないから話せないんだよ

分かったところで話せないらしいぞw

662 :名無しさん@英語勉強中:2015/12/12(土) 10:54:24.45 ID:alpdtTmW.net
ま英文法は、中学・高校生には難しいかもしれないけど、
大学生以上は誰でも知ってるし、本開けば誰でも解説できる。
その程度のこと。現代国語や社会科科目と変わらん。
自慢にはならんな

663 :名無しさん@英語勉強中:2015/12/12(土) 14:52:57.24 ID:24Z878+F.net
>>633
アインシュタインは宇宙飛行士でもなく宇宙旅行をしたこともないのに
宇宙の法則に関する理論を組み立ててしまいましたけどね。
彼が特に実験家であったわけでもない。理論の中で彼はそれをやり遂げた。
これが近代科学の成果ですよ。理論をバカにしちゃいけません。

一経営者や一労働者よりも経済学者のほうが経済について知っていることもあるでしょう。
経済学者が知らないことも多いでしょうが、一面の真理だけを振りかざして他者の活動を
上から目線で裁断しようとするのは自己中心的で偏見に満ちた考え方というものですよ。

664 :名無しさん@英語勉強中:2015/12/12(土) 16:42:15.10 ID:LNzULO6a.net
英語に文法は要らないよ
いつまで前時代的な学習をしてるんだ?

665 :名無しさん@英語勉強中:2015/12/12(土) 17:30:04.77 ID:lOqOG/ha.net
文法が要らないなら、「私を連れて行ったんだ、彼は」を
Me took he.とやればいいわけですね?

666 :名無しさん@英語勉強中:2015/12/12(土) 18:17:15.64 ID:AHH2lpsI.net
日本語大杉
英語コンプの荒らしはやめ

667 :名無しさん@英語勉強中:2015/12/12(土) 18:47:01.72 ID:gkebS7Qy.net
And that is why the confidence game is both the oldest there is and the last one that will still be standing when all other professions have faded away

この文の構造がわかりません 解説お願いします

668 :名無しさん@英語勉強中:2015/12/12(土) 19:08:32.22 ID:33tmrhkX.net
>>663
英語は言葉の習慣にすぎないから、
自然科学の定理の援用みたいな考え方で文法を扱っても
はうまく行かないよ

そんなことしてたら一生話せないまま終わる

669 :名無しさん@英語勉強中:2015/12/12(土) 19:12:47.65 ID:c6/72HCU.net
>>667
>And that is why the confidence game is both the oldest there is and the last one that will still be standing when all other professions have faded away

そして、それがconfidence gameが、存在する最も古いものであると同時に、他の全てのprofessionsがfade awayしても、最後まで残る理由である。

670 :名無しさん@英語勉強中:2015/12/12(土) 19:13:26.17 ID:7hHoMPJi.net
文法否定してるやつで、文法学習経験ゼロなやつがいないのがね。

671 :えワ:2015/12/12(土) 19:15:02.99 ID:MqYG+ov/.net
>>667
出典:ニューヨークタイムズ
タイトル: Born to Be Conned 
筆者:MARIA KONNIKOVADEC. 5, 2015

まぁ、チンコピアスが自演で答えるんだろう。

672 :名無しさん@英語勉強中:2015/12/12(土) 19:25:45.35 ID:A8rl6/D9.net
>>667
別の人が、きれいに和訳してくれた。

And that is why the confidence game is
  BOTH
    (1) the oldest [one that] there is
  AND
    (2) the last one that will still be standing (= 「存続している」という意味の形容詞)
      when all other professions have faded away

673 :名無しさん@英語勉強中:2015/12/12(土) 20:11:05.43 ID:EBHNTndY.net
X-MENの予告映像見てて思ったんですが、
1分34秒の「THIS SUMMER」って
2015年夏のことなんですか?
2016年夏のことなんですか?
https://youtu.be/COvnHv42T-A?t=93

674 :名無しさん@英語勉強中:2015/12/12(土) 20:27:19.13 ID:A8rl6/D9.net
>>673
この映画のストーリーを知らないけど、この英語字幕だけを見て判断すると、

  This summer, prepare for the apocalypse.

というふうにつながっているから、やはり 2016 年の夏のことでしょう。
まさか過去の夏に戻って「Prepare せよ」などと命令形を使うはずもないと思います。

675 :名無しさん@英語勉強中:2015/12/12(土) 20:28:07.44 ID:EBHNTndY.net
>>674
ありがとうございます!

676 :名無しさん@英語勉強中:2015/12/13(日) 00:01:01.97 ID:hA5QepCQ.net
現在ではなく過去のことに関して、ネイティブ・スピーカーの方々は
たとえば「If i hadn't〜」ではなく「If i didn't〜」ということが多々あるように思うのですが
これは彼らの間では普通のことなのでしょうか?

677 :えワ:2015/12/13(日) 00:10:21.83 ID:PhZua64x.net
>>676
時制で、特別な事情がなければ、「単純過去の表現へ」という英語の流れはある。

678 :名無しさん@英語勉強中:2015/12/13(日) 00:12:42.33 ID:E5CW4hrt.net
過去のことを仮定するときは必ず仮定法じゃないといけないわけじゃないから
hadn'tなら仮定法、didn'tなら仮定法じゃない

679 :名無しさん@英語勉強中:2015/12/13(日) 03:55:37.44 ID:UFoK4UuY.net
>>676
直接法過去だね。
If he didn't read it, it's natural he doesn't know.
彼がそれを読んでいなかったのなら、知らないのも当然だ。
事実に反するわけじゃないから仮定法過去完了にはならないね。

680 :名無しさん@英語勉強中:2015/12/13(日) 08:37:59.58 ID:mLObd/Nd.net
直説法なw

681 :名無しさん@英語勉強中:2015/12/13(日) 10:47:19.83 ID:wCARSKip.net
English composition

I think it depends on how old you are whether grammar is necessary or not for mastering English.
With regard to acquiring languages, there is a pypothesis called " rinkaiki kasetsu."
According to this pypothesis, if you are under arond the age of ten, you can master a second launguage without grammar like native speakers of the laungage do.

682 :名無しさん@英語勉強中:2015/12/13(日) 10:49:39.40 ID:wCARSKip.net
typo popothesis → hypothesis

683 :えワ:2015/12/13(日) 11:02:51.68 ID:PhZua64x.net
臨界期仮説(りんかいきかせつ、英: critical period hypotheses)

684 :名無しさん@英語勉強中:2015/12/13(日) 11:31:25.90 ID:zcveZSSG.net
However the native text input systems is not as attractive as I like so I thought I would look at (B社の) features.

すいませんこの文の訳がよくわからないのですが何と言ってるのでしょうか?

685 :えワ:2015/12/13(日) 11:43:53.68 ID:PhZua64x.net
>>684
B社の製品の方がいい。私にとって使いやすい、ぐらいかな。

686 :名無しさん@英語勉強中:2015/12/13(日) 11:46:32.15 ID:s2UTGpVx.net
hopes are high that a solution to their frequent disputes has been worked out.

thatが接続詞でhopesの内容を表しているらしいけど、
こんな構文なりたつんでしょうか。

「are high」のような言葉が間に入る文章を見た覚えがないものでして。
どなたかご教示いただけると助かります。よろしくお願いいたします。

687 :名無しさん@英語勉強中:2015/12/13(日) 12:00:04.87 ID:zcveZSSG.net
>>685
ありがとうございます。

688 :名無しさん@英語勉強中:2015/12/13(日) 12:02:24.57 ID:mLObd/Nd.net
同格のthat以下が長すぎるから後に置いてるだけ

689 :名無しさん@英語勉強中:2015/12/13(日) 12:30:16.55 ID:/GdMuntJ.net
犯罪の被害者の死にたいとまで考える被害者の精神的苦痛、悲嘆と無念を
自分自身がそうした犯罪被害にあったものとして(自分自身のものとして)、あるいは自分の親、家族、恋人が同じような犯罪の被害にあったり、犠牲になったものとして
考えることはできますか?

というのを英語で訳すとどうなるのでしょうか?

690 :名無しさん@英語勉強中:2015/12/13(日) 13:32:24.00 ID:creZBLth!.net
アメリカのレストランで食事をした際の請求書に1割ほどのgratitudeというのが入っていたから、それがチップだろうと思って、クレジットカードを渡したら、
客控えと店控え付きで返ってきて、そこにもチップとtotalの空白欄があった。

チップとgratitudeとは別なのかなと思って、チップと合計の金額を書いて、一緒に再度クレジットカードと渡したら、もう会計は終わってると呆れられたんだけど、これって何がどういうことだったのか、わかる人いますか?

691 :名無しさん@英語勉強中:2015/12/13(日) 13:37:02.69 ID:DtMp1LLD.net
>>686
Hopes are high that S + V.
  = Hopes that S + V are high.

こういうのは、よく出てきますよ。その他の例。

(1) Expectations are high that S + V.
  While they don't charge for their expertise,
  ●the expectations are high that● you'll buy at least some products.

(2) Demand is high for AAA.
  Real estate agents say ●demand is high for● townhomes and
  patio homes in the Inner Loop.

(3) Demand is growing for BBB.
  In addition, ●demand is growing for● business investments
  to support equitable development.

以上の例文の出典は、Corpus of Contemporary American English。
他にもいろいろありうるけど、ちょっと検索が難しい。また思いついたら、書き込みます。

692 :名無しさん@英語勉強中:2015/12/13(日) 13:54:28.84 ID:DtMp1LLD.net
>>689
犯罪の被害者の死にたいとまで考える被害者の精神的苦痛、悲嘆と無念を
  When you see how much the victims of a crime suffer,
  what pains, griefs, and regrets they go through, so much so that
  they even think of killing themselves,

自分自身がそうした犯罪被害にあったものとして(自分自身のものとして)、
  can you emphathize with them as if you yourself
  had been victimized in the same way,

あるいは自分の親、家族、恋人が同じような犯罪の被害にあったり、
犠牲になったものとして 考えることはできますか?
  or as if your parent, family, or lover were the victim
  of a similar crime?

693 :名無しさん@英語勉強中:2015/12/13(日) 15:53:20.61 ID:iBEKPPRW.net
My decision to drive on turned out to be right

この1文の動詞ってturned outだと思うのですがそれだとdrive onのonという前置詞の
目的語はなぜないんでしょうか

694 :名無しさん@英語勉強中:2015/12/13(日) 16:14:17.24 ID:IZy2baXd.net
>>693
drive onで「運転し続ける」という句動詞だからです。
ちなみにこのonは副詞です。

695 :えワ:2015/12/13(日) 16:34:50.49 ID:PhZua64x.net
>>690
ググってみたら、こんなアンケートが見つかった、gratitude = compulsory charge は「強制的に請求されるチップ」みたいだね。
「gratitude が料金に含まれているから、ウェーターにはチップは渡さないで」という表記がメニューにあったのかもね。

1. Nowadays many restaurants automatically added 15 -20 % of gratitude to your bill,
what do you think about this compulsory charge?
最近、請求書に15〜20%の gratitude を自動的に請求するレストランがありますが、どう思いますか。

アンケートの答え、つまり認識はこんな感じ。

- チップはいいサービスに対して支払うもので、強制的に取るならサービスが落ちる 68人 26%
- チップは店じゃなく、個人にあげたい。 115人 44%
- そもそも、チップの制度が嫌い。それもコストとして、料金に入れておけ。 34人 13%
- 次は、銀行からマクドナルドまで、全部で手数料として取り始めるかも。 23人 8%
- いいね。計算しなくていい。 15人 6%
- その他 5人

696 :名無しさん@英語勉強中:2015/12/13(日) 19:12:26.58 ID:wVgpPU+S.net
歌詞です
このeverってあるとないではどう違うんでしょうか?

Ever close your eyes
目を閉じて
Ever stop and listen
耳を澄ませば
Ever feel alive
命を感じる

697 :名無しさん@英語勉強中:2015/12/13(日) 19:24:08.31 ID:mLObd/Nd.net
辞書引け

698 :名無しさん@英語勉強中:2015/12/13(日) 19:30:21.15 ID:X01aG8cY.net
普通は使わないね。

699 :名無しさん@英語勉強中:2015/12/13(日) 19:37:34.45 ID:DtMp1LLD.net
>>696
僕が思うには、これら三つのフレーズは、最初に Do you をつけて疑問文に
するはずだったものと思う。ever をつけると「一度でも」というような意味になる。
Do you close? だったら「ふだん〜を閉じることがあるか?」というような
現在の習慣を表す。ever がついているから、「ふだん、一度でも〜することはある?」
という問いかけになる。

(1) (Do you) ever close your eyes?
  (ふだん、一度でも)目を閉じることはある?
(2) (Do you) ever stop and listen?
  (ふだん、一度でも)立ち止まって耳をすますことがある?
(3) (Do you) ever feel alive?
  (ふだん、いちどでも)生きてるっていう実感を感じることある?

700 :名無しさん@英語勉強中:2015/12/13(日) 19:38:14.96 ID:pDPIwy9M.net
we’ll show you that the epsilon-Greedy algorithm can also be configured to behave
exactly like the profit-maximization rule that Bob hoped Deb would settle upon after
her experimental phase was over.
このbob hopedってやつは連鎖関係代名詞というやつですか?

701 :名無しさん@英語勉強中:2015/12/13(日) 19:56:27.84 ID:wVgpPU+S.net
>>699
ありがとうございました
分かりやすかったです

702 :名無しさん@英語勉強中:2015/12/13(日) 20:05:21.78 ID:E2zRgAYT.net
初歩の文で申し訳ないけど、オスカーワイルド、幸福の王子の
High above the cityってどういう文法構造になってるの?
highは副詞で、above the cityはhighを修飾してると解釈すればいいの?
前置詞は副詞を修飾できるの?

703 :名無しさん@英語勉強中:2015/12/13(日) 20:17:37.60 ID:6LMQ2tCd.net
accompanyについて
His wife accompany him.
「彼は夫人を同伴した」が正解で「夫人が彼を同伴した」は誤りと習いました。

そこで、
A accompanying B の場合、ついていくのはAとBのどちらですか?

704 :名無しさん@英語勉強中:2015/12/13(日) 20:20:47.01 ID:VteT1f3I.net
A

705 :名無しさん@英語勉強中:2015/12/13(日) 20:22:12.51 ID:mLObd/Nd.net
辞書引け

706 :名無しさん@英語勉強中:2015/12/13(日) 20:27:14.78 ID:DtMp1LLD.net
>>703
次のリンク先を読めばわかる。こんなことはどんな辞書でも載っているはず。

  http://ejje.weblio.jp/content/accompany

707 :名無しさん@英語勉強中:2015/12/13(日) 20:35:26.20 ID:DtMp1LLD.net
>>702
Oscar Wilde は好きだし、特に "The Happy Prince" が好きなので、答えたくなった。

●High above the city●, on a tall column, stood the statue of the
Happy Prince.

"above the city" なら、「その町の上に」という意味。high がつくと、
「その町のはるか上の方に」という感じ。自然な感じに訳すと、
「その町の地上からはるか空高く」という感じかな?
文法用語なんか忘れて、ともかくどんどん読み進めることだ。70% くらい理解できた
と思ったら、どんどん先を読み進める。そして、1,000ページほど読み進めたころに、
英文をけっこう読みなれていることに気づく。そしてその調子で、1万ページを読み進める。
そのあと、またやる気が出てきたら、3万ページほど読む。

708 :703:2015/12/13(日) 20:44:25.25 ID:6LMQ2tCd.net
passengers accompanying children
という文はあやまりですよね?

709 :703:2015/12/13(日) 20:45:49.74 ID:6LMQ2tCd.net

車内放送で「子供連れの乗客は」といいたいとき

710 :名無しさん@英語勉強中:2015/12/13(日) 20:46:50.56 ID:E2zRgAYT.net
>>707
何詞とかこまけぇこたぁいいんで、とにかくhighがゆるくabove以下を修飾してると考えればいいのでしょうか
辞書的な用法としてはこれに当てはまるのでしょうか

【副詞】

1



非常に高い高度で

(at a great altitude)




he climbed high on the ladder 彼は梯子で高く上った


お答えいただきありがとうございました

711 :名無しさん@英語勉強中:2015/12/13(日) 21:08:08.51 ID:DtMp1LLD.net
>>710
あなたの引用したその辞書の例文も参考にしていいと思いますね。もう一度、
Oscar Wilde の文章に戻ってみます。

●High above the city●,
  その町の地上から空高く、
on a tall column,
  背の高い円柱の上に、
stood the statue of the Happy Prince.
  幸福な王子の彫像が立っていた。

この文は倒置されていますが、普通のわかりやすい文に変えてみます。

The statue of the Happy Prince
  幸福な王子の彫像が、
stood high above the city, on a tall column.
  背の高い円柱の上に、その町からずっと空高く立っていた。

このような "high above the city" によく似た言い回しがたくさんありますが、
たとえば The airplane was flying high in the sky. という言い回しもあります。

He saw himself in the seat of an airplane flying ●high in the sky●,
  (Google Books というサイトで見つけた文章)

712 :名無しさん@英語勉強中:2015/12/13(日) 21:09:43.50 ID:Dgn+YrD7.net
>>702
日本語でも作品名だと修飾語だけで被修飾語が存在しないことってあるでしょ
「北の国から」とか「てぶくろをかいに」とかさ

713 :えワ:2015/12/13(日) 21:10:44.13 ID:PhZua64x.net
>>701
お前、馬鹿だろう。
せめて、Enya の Wild Child ぐらい書きなよ。

と、見たら、チンコピアスの自演か。

714 :名無しさん@英語勉強中:2015/12/13(日) 21:40:22.24 ID:E2zRgAYT.net
>>711
さらに詳しくありがとうございます


"stood" "high" "above"それぞれの修飾関係は

1)
stood←high

above

2)
stood←high←above

3)
stood←above←high

上のようにいくつかのパターンが考えられて、それで直感的に文意が取りづらくて質問させていただきましたが、
それが分かったところで、だからなんだという感じですし、
自然言語の文法は後付けでしかないのだから、言い回しとして覚えたほうがいいですよね
それはともかく、挙げていただいた例文も含めて理解が深まり、勉強になりました
ありがとうございました

715 :名無しさん@英語勉強中:2015/12/13(日) 22:22:58.61 ID:77OGQ5tn.net
久しぶりにみたが、このえワ成りすまし?

基地外のキレが偽者臭く、池沼感がない。
ガチ池沼の振りするのはさすがに無理か

716 :名無しさん@英語勉強中:2015/12/13(日) 22:23:14.38 ID:tr/UvKzM.net
to do is to V か to do is V
to do is Vについての質問がどっかで出てた気がしたが
今日これを読んだので参考に紹介しとく
現代英語ではto do is Vが優勢

http://languagelog.ldc.upenn.edu/nll/?p=22784#more-22784

717 :686:2015/12/13(日) 23:19:43.14 ID:s2UTGpVx.net
>>686  

同格のthatの後置でしたか。ありがとうございます。

>>689

他の例文まであげて頂き、非常に参考になります。
お手数をお掛けしました。ありがとうございます。

718 :686:2015/12/13(日) 23:21:45.02 ID:s2UTGpVx.net
↑の書き込み

>>686ではなくて >>688です。
失礼しました。

719 :名無しさん@英語勉強中:2015/12/14(月) 00:15:56.66 ID:P0s6FQzQ.net
>>669 672
ありがとう

720 :えワ:2015/12/14(月) 00:18:07.35 ID:Iw5Jpj2z.net
>>718
お手並みを拝見しようか。

721 :えワ:2015/12/14(月) 02:01:13.55 ID:2C0raep2.net
>>720

 *'``・* ↓ID:Iw5Jpj2z = チンコピアス
    |     `*
   ,∩∧__∧  *   
  +..<,,`∀´,,> *+ えワにな〜れ
  `* ヽ、 ⊂ **
   `・+*・' +⊃
  ☆  ∪~ ~ **
    `・+*・

           *
     ∧_,,∧  |
    < `∀´;>つ   ・・・・・・・
     (つ   /   ・・・
      し⌒ J    なれなかったみたい・・・・・


         *
    ..∧_,,∧|
    <∩Д∩>     シクシク
    (     )
      し⌒ J

722 :えワ:2015/12/14(月) 03:31:45.28 ID:2C0raep2.net
>>699
every っていう単語を知ってる?

723 :名無しさん@英語勉強中:2015/12/14(月) 03:36:31.90 ID:qTs8DKJ4.net
「please be excited」って英語の文法として間違ってますか?
検索するとスクエアエニックスのプロデューサーの発言がミーム化してます

https://www.google.com/search?q=please+be+excited

文法が間違ってるとしても、日本人が英語を間違えるなんてよくあることだと思いますが
これって間違え方が面白いんでしょうか

724 :えワ:2015/12/14(月) 04:01:15.24 ID:2C0raep2.net
>>723
他の表現はあるけど、文法としては、間違ってはいないと思う。
ミーム化してるのは、普通にゲームが注目されてるからだろうね。
個人的には「粋な表現」だと思う。

Enix will excite you 「楽しんで下さい」
→「楽しんじゃって下さい」ぐらいの感じになるかな。

725 :名無しさん@英語勉強中:2015/12/14(月) 04:11:31.49 ID:qTs8DKJ4.net
>>724
そうなんですか
ありがとうございます

726 :えワ:2015/12/14(月) 04:16:25.45 ID:2C0raep2.net
beloved, bewitched あたりの単語を調べてみるのもいいだろうね。
日本人には使いこなせない表現だから。

727 :名無しさん@英語勉強中:2015/12/14(月) 04:24:45.66 ID:qTs8DKJ4.net
ありがとうございます

728 :えワ:2015/12/14(月) 06:35:59.34 ID:Iw5Jpj2z.net
>>726
まあ、チョンコに使いこなすのも難しいだろうけどね。

729 :名無しさん@そうだ選挙に行こう:2015/12/14(月) 08:05:42.92 ID:Q+eTft3I.net
小説や漫画でみるようなキャラクターの二つ名について質問です。
英語で二つ名を書くときの規則性のようなものはありますか?

・緑葉のレゴラス "Legolas the Greenleaf"
・疾風迅雷のリロイ "Leroy the Lightning speed"
・冷血のレナ "Lena the Cold blood"

最初は上記のように 名前 冠詞 二つ名 の順番を小説や映画で見かけたので
こういう法則性かふむふむと思ったものですが

・悪魔の子 ニコ・ロビン "Devil Child Nico Robin"
・点火するものデアリガズ "Darigaaz, the Igniter"

上記のような英語表記もみかけたりで法則性がよくわかりません。
どういう使い分けがあるんですか?

730 :名無しさん@そうだ選挙に行こう:2015/12/14(月) 08:30:33.29 ID:m5JkDfaK.net
「大きな古時計」の元歌の

731 :名無しさん@そうだ選挙に行こう:2015/12/14(月) 08:30:58.92 ID:NQtynZiI.net
This is one reason confidence games flourish, why anyone, no matter how honest, is a potential victim

これは one reason that(同格) confidence games flourish
one reason why(関係副詞) anyone is a potential victim
でしょうか?

732 :名無しさん@そうだ選挙に行こう:2015/12/14(月) 08:34:25.57 ID:m5JkDfaK.net
すみません、書き込みに失敗しました。

「大きな古時計」の元歌の "Grandfather's Clock"の歌詞で
日本語で「今はもう動かない」に当たる部分が
多くは "But it stopped short" となっているのですが、少数のサイトで
"But it stop'd short" となっています。
これは古い英語なのでしょうか?また、略さずに書くとどのようになるのでしょうか?

733 :名無しさん@そうだ選挙に行こう:2015/12/14(月) 09:14:06.60 ID:v8Em8WIW.net
>>731
両方とも 関係副詞だと思うけどね。前の方は自明だから省略されている。
つまり二つの関係副詞節がone reason という先行詞に掛かっていると解釈できる。

734 :名無しさん@そうだ選挙に行こう:2015/12/14(月) 09:15:42.32 ID:v8Em8WIW.net
why が省略

735 :名無しさん@そうだ選挙に行こう:2015/12/14(月) 09:25:13.57 ID:xIdlJmMI.net
>>732
歌詞はもともと詩ですが、昔の(おそらく19世紀くらいまでの)詩においては、
stopped と書くと [stɑpid] または [stɔpid] と発音し、stop'd または stop't
と書いて初めて [stɑpt] または [stɔpt] という発音が表せたのです。
たとえば Shakespeare の "Romeo and Juliet" の冒頭には
  The fearful passage of their death-mark'd love,
と書いてあり、mark'd と書くことによって [mɑːkt] という発音を表し、もしも
これを marked と書いてしまうと、詩においては [mɑːkt] と発音しなければならないことになって
いたのです。このような名残を残して、今回の Grandfather's Clock においても
stop'd と書いていたのかもしれません。

もう一つの理由としては、この歌詞を書いたのは 1876 年のアメリカ人ですが、
そのころ、stopped の代わりに stopt または stop't と
書く人がいたのかもしれません。OED という20巻にわたる大きな辞書には、
次のように stop't を使った例文が載っています。ただしこれは、18世紀ころに
すたれてしまった綴りであり、現代では使われていないようです。

1697 Dryden tr. Virgil Æneis ii, in tr. Virgil Wks. 261,
  I went; but sad Creusa ●stop'd● my way, And cross the Threshold
  in my Passage lay.

1708 London Gaz. No. 4431/15
  They have anchor'd and ●stop'd● the Tide.

   (OED Second Edition より)

736 :名無しさん@そうだ選挙に行こう:2015/12/14(月) 09:28:11.42 ID:xIdlJmMI.net
>>735
書き間違いました。訂正します。【  】の中が、訂正部分です。
 
   これを marked と書いてしまうと、詩においては 【[mɑːkid]】 と発音しなければならないことになって

737 :名無しさん@そうだ選挙に行こう:2015/12/14(月) 09:41:42.36 ID:xIdlJmMI.net
>>731
同格ではなく、関係副詞の why が that で代用され、そのあと that が省略された
のです。そのことは、たとえば「ジーニアス英和、第5版」の「代名詞としての that の
(2)」の項目で、10行くらいにわたって解説されています。

738 :名無しさん@そうだ選挙に行こう:2015/12/14(月) 09:46:32.57 ID:xIdlJmMI.net
>>729
それについて詳しく知りたければ、次の本の該当箇所を読んでみてください。その
問題だけのために、なんと4ページもの解説があります。

   現代英語冠詞事典、樋口昌幸著、pp.279-282

739 :名無しさん@そうだ選挙に行こう:2015/12/14(月) 10:42:46.88 ID:x7+nUlLw.net
英語を学ぶ、ではなく、日本語を学ぶ外国人に関わる相談なのですが、お願いします。

カナダ在住の、1年ほど独学で日本語を学んでいる女性(成人で社会人)にオススメの日本語文法のテキストはないかと聞かれました。
「これは良い文法書?」とアマゾンで見せられた本は、自分が見ても難解で例文等もいいものではないと思いました。
ちなみに私は日本在住です。おたがいカタコトで雑談程度のやりとりで、私自身が彼女に日本語をちゃんと指導はできません。
あまりぶ厚くて高価な本は敬遠しているようなので、手頃な価格でわかりやすい日本語文法の本はないでしょうか。
カナダのアマゾンで注文できればよいのですが……。

740 :名無しさん@そうだ選挙に行こう:2015/12/14(月) 10:45:12.57 ID:1YcM0ilz.net
板違いだろ

741 :名無しさん@そうだ選挙に行こう:2015/12/14(月) 10:45:52.79 ID:mlbq4Kd1.net
>>738
やっぱその本は英語やるなら必須だよな

742 :名無しさん@そうだ選挙に行こう:2015/12/14(月) 12:02:20.28 ID:WAYYgpfx.net
>>700
がスルーされてるような気がするんですが?
気のせいですか?

743 :名無しさん@そうだ選挙に行こう:2015/12/14(月) 12:16:29.77 ID:xIdlJmMI.net
>>700
悪気で無視していたのではなく、この質問が理科系の文章に関するものだから、
あまり下手な解説をしても無駄だろうと思って、敬遠していたのです。

原文:
we’ll show you that the epsilon-Greedy algorithm can also be configured to behave
exactly like the profit-maximization rule that Bob hoped Deb would settle upon after
her experimental phase was over.

連鎖関係代名詞かどうかは知りませんが、ともかく settle upon のあとに本来なら
the profit-maximization rule が来るはずなのです。つまり、

   Bob hoped [that] Deb would settle upon 【the profit-maximization rule】
   after her experimental phase was over.

という文が頭の中にあるのだと考えればいいと思います。理科の専門知識がないので、
僕がとんちんかんな解釈をしていなければいいのですが。

744 :名無しさん@そうだ選挙に行こう:2015/12/14(月) 12:30:22.40 ID:fI2z6WgP.net
>>739
リンク>>222の3番目の動画みてもらうといいよ。カナダ出身の女性のアドバイスなので

745 :三年英太郎 ◆3CZBjOt3.Y :2015/12/14(月) 13:05:44.63 ID:uOMzdOW8.net
>>739
ひとりでできる 初級日本語文法の復習 英語版
http://www.3anet.co.jp/ja/1238/

この出版社は、日本語教育の超定番教科書を出してるところなので、
内容については信頼できる。

日本のアマゾンを英語表記に切り替えて、海外発送してもらえばいい。

746 :三年英太郎 ◆3CZBjOt3.Y :2015/12/14(月) 13:08:05.97 ID:uOMzdOW8.net
英語リンク
http://www.amazon.co.jp/gp/switch-language/product/4883195414/ref=dp_change_lang?ie=UTF8&language=en_JP

747 :名無しさん@そうだ選挙に行こう:2015/12/14(月) 13:10:47.80 ID:WAYYgpfx.net
>>743
ボブは今すぐプロフィットマキシマイゼーションルールを使いたいと思っているようなので
違うと思います
それではないとしたら連鎖関係代名詞ですよね

748 :名無しさん@そうだ選挙に行こう:2015/12/14(月) 13:16:09.33 ID:xIdlJmMI.net
>>747
>>ボブは今すぐプロフィットマキシマイゼーションルールを使いたいと思っている

そのような状況と、僕の解釈とは、まったく矛盾してはいないはずだけど。
なぜおかしいと思うのかな?「連鎖関係代名詞」なんていう気持ちの悪い
用語を使わず、あなたの解釈で和訳してみてはどうかな?

749 :名無しさん@そうだ選挙に行こう:2015/12/14(月) 13:32:09.26 ID:WAYYgpfx.net
ボブは彼女の経験フェーズがすんだ後でDebがプロフィットマキシマイゼーションルールを取り決めてほしいと思っているのだから
今すぐじゃないですよね
ボブは経験フェーズなんていうものはいらないという意見の持ち主なので

750 :名無しさん@そうだ選挙に行こう:2015/12/14(月) 13:35:58.83 ID:xIdlJmMI.net
>>749
でも英文にはそのように書いてあるのだから、仕方がないでしょう?
他にどういう解釈があるのですか?もしかして「連鎖関係代名詞」
とさえ言えば、わからないものがわかるとでも思っているのですか?
その文法用語の本当の意味を知っているのですか?

751 :名無しさん@そうだ選挙に行こう:2015/12/14(月) 14:07:40.15 ID:xIdlJmMI.net
>>749
僕は「連鎖関係代名詞」というものを一応は理解しているつもりだけど、それを
適用してもやはり僕の言う通りの解釈しか成立しないと思う。あなたの言う
「連鎖関係代名詞」がどういうものか知らないけど、そのあなたの解釈の仕方で
この構文を説明してください。他の解釈があるなら、ぜひ知りたいので。

752 :えワ:2015/12/14(月) 14:07:54.84 ID:2C0raep2.net
>>750
自演はいい加減にしなよ。

753 :名無しさん@そうだ選挙に行こう:2015/12/14(月) 15:49:14.72 ID:WAYYgpfx.net
profit-maximization rule that Bob hoped 【the profit-maximization rule and】 Deb would settle 【the profit-maximization rule and】 upon after
her experimental phase was over.
こんな感じだとおもうんですけど

754 :名無しさん@そうだ選挙に行こう:2015/12/14(月) 15:55:24.79 ID:xIdlJmMI.net
>>753
それは英文ではなくなってしまっていると思いますよ。いったいどういう意味になるのですか?
もちろん、nonnative が書いた英文ならどんな英文でもありえますが、あくまで正しい
英文だと仮定すると、どういう意味になるのですか?

755 :名無しさん@そうだ選挙に行こう:2015/12/14(月) 16:02:48.44 ID:WAYYgpfx.net
ボブが望んだ、そして
デブが経験フェーズの後に取り決めるだろうとする
プロフィットマキシマイゼーションルール
これなら文脈と一致しますね
これにあう文法が連鎖関係代名詞でしか説明が付きませんね

756 :えワ:2015/12/14(月) 16:06:36.58 ID:2C0raep2.net
>>754
自演はいい加減にしなよ。
英文のレベルと質問のレベルが違うんだよ、バレバレだ。
まだ、やるの?

757 :名無しさん@そうだ選挙に行こう:2015/12/14(月) 16:56:27.87 ID:BUMvW3B9.net
初歩的なことですみません、この例文のby takingのところがさっぱりわかりません
to take outやbyを消してtaking outではだめなのでしょうか?

he made a fortune by taking out a patent on a new way to manufacture license plate

758 :名無しさん@そうだ選挙に行こう:2015/12/14(月) 16:59:19.79 ID:6nUDbB2M.net
ダメです

759 :名無しさん@そうだ選挙に行こう:2015/12/14(月) 17:04:33.46 ID:6nUDbB2M.net
不定詞は文法書で名詞の後の不定詞の用法を参照
byを取るとS+Vの関係になるので意味不明になる

760 :名無しさん@そうだ選挙に行こう:2015/12/14(月) 17:10:40.31 ID:BUMvW3B9.net
>>759
ありがとうございます

761 :名無しさん@英語勉強中:2015/12/14(月) 20:12:24.82 ID:o1iKd/nR.net
「ダウンタウンのガキの使い罰ゲームは今年で終わるという噂を聞いています」

答え
白人女性「that's sucks!」
これはどういう意味になりますか?「マジで!」くらいの言葉でしょうか?
suckって相当ヤバイ言葉かと思ったんですが
普通にびっくりした時にも使うんでしょうか?

762 :名無しさん@英語勉強中:2015/12/14(月) 20:31:14.75 ID:m5JkDfaK.net
>>732です。
ありがとうございました。

763 :えワ:2015/12/14(月) 20:55:01.56 ID:2C0raep2.net
>>761
suck の基本的な意味は「口で吸う」だけど、「同意出来ない」という意味もある。
同意できない、納得出来ない:「エーッ、何それ、信じられない、何でよぉ、何考えてるの」あたりかな。

こんなイディオムがあった。「ヤブー サック!」
Yah boo sucks - Yah boo sucks can be used to show that you have no sympathy with someone.
全く同意出来ない事を示す表現。「好きにすれば、勝手にすれば、私関係ないから。」ぐらいかな。

764 :777 ◆TFWBMdHdF7zL :2015/12/14(月) 21:28:22.02 ID:2FIzKREq.net
>>763
>同意できない、納得出来ない:「エーッ、何それ、信じられない、何でよぉ、何考えてるの」あたりかな。

ちょっと違う。
辞書引けばいいだけじゃん。

http://www.oxforddictionaries.com/definition/english/suck
[no object] (North American informal) Be very bad or unpleasant

http://ejje.weblio.jp/content/suck
《米俗》〈事が〉いや気がさす,むかつく.

765 :えワ:2015/12/14(月) 21:57:42.27 ID:2C0raep2.net
>>764
日本語は苦手?
質問者に答えてあげてね、偽777ことチンコピアス君。
コテを変えてもね、馬鹿はどうしようもないんだよ。

766 :名無しさん@英語勉強中:2015/12/14(月) 23:22:24.35 ID:o1iKd/nR.net
>>763
なるほど
容認出来ないという感じなんでしょうね
ダウンタウンが好きらしいので
ありがとうございました

767 :名無しさん@英語勉強中:2015/12/15(火) 09:34:20.25 ID:0Pfv1yQx.net
>>733 >>737
ありがとう

768 :名無しさん@英語勉強中:2015/12/15(火) 10:39:24.26 ID:6td/bfLb.net
declined by over two thirds
という文なのですが、by は差を表すと思いますので、
この文は3分の2以上が減った、つまり、
3分の1以下になってしまったという意味だと
解釈しているのですが、正しいでしょうか?

769 :名無しさん@英語勉強中:2015/12/15(火) 11:06:42.46 ID:5gGviDyS.net
>>768
あなたの言う通り、間違いありません。そのことを数字ではっきりと証拠づけてくれている
例文が2つ見つかりました。数字が確かに 3/2 以上減って、結果として 1/3 以下に
なっていることが、次の例文を見ればわかります。

(1) The number of vehicle donations actually ●declined by
over two-thirds●, from 900,000 in 2004 to under 300,000 in 2005.

(2) Revenue per barrel of oil equivalent (BOE) from oilsands
●declined by over two-thirds●, from $1.60 in 1995 to 50 cents in 2004.

(3) As a percentage of the total area cut, clearcutting
●declined by over two thirds●, from 39 to 12 percent.

出典:
https://www.google.co.jp/webhp?sourceid=chrome-instant&ion=1&espv=2&ie=UTF-8#q=%22declined%20by%20over%20two%20thirds%20from%20*%20to%22

770 :名無しさん@英語勉強中:2015/12/15(火) 11:20:09.24 ID:0Pfv1yQx.net
Toyota Motor Corp. earned an unprecedented \827.4 billion in the quarter ended in September, more than twice what General Motors Co. and Ford Motor Co. posted combined.

what General Motors Co. and Ford Motor Co. posted combined.

General Motors Co. and Ford Motor Coを組み合わせて、元帳に記載したことがっていうのはわかるんですが

英語的に構造がわかりません 解説お願いします

771 :768:2015/12/15(火) 11:32:58.37 ID:6td/bfLb.net
>>769
教えて頂いてありがとうございます。
Web上で実際の例文も挙げて頂き、
非常に参考になりました。
「by 数字」は差を表すという文法知識は
頭では知っていたものの、日本語では
「3分の2以上の減少量をもって減少した」
という言い方はほとんど聞かないため、
英語では日本語と違ってそのような言い方も
するのだろうか、それとも私の解釈が間違って
いるのだろうかと不安に感じていました。
実際の数字が入った例文を提示して頂き、
すっきり疑問が解決しました。本当にありがとうございました。

772 :名無しさん@英語勉強中:2015/12/15(火) 11:58:46.91 ID:5gGviDyS.net
>>770
(1) >>what General Motors Co. and Ford Motor Co. posted combined.

質問は、上記のフレーズについてのみですね?combined という単語は、ご存じでしょうが
次のように使われますね。

    The bottom line is clear: The U.S. creates far more new jobs
   (net of layoffs) than ●Europe and Japan combined●.
     (Corpus of Contemporary American English より)

上記の例文の大意は、アメリカで創出された新規の雇用口の数は、
●欧州のそれと日本のそれとの合計● をはるかに上回っているという意味ですね。

それと同じように、今回のフレーズである上記の (1) は、
「GM の収益と Ford の収益との合計」という意味ですね。このフレーズの
構造は、
  what General Motors Co. earned and what Ford Motor Co. earned combined
を省略して表現したものだと思います、つまり "A and B combined" という形になって
いて、上記の corpus における例文の中での "Europe and Japan combined" と
まったく同じ構造をしているのです。

773 :名無しさん@英語勉強中:2015/12/15(火) 12:06:35.79 ID:5gGviDyS.net
>>772 にて書き間違いをしました。正しく書き直した部分を【   】で示します。

   what General Motors Co. earned and what Ford Motor Co. 【posted】 combined

上記の posted はご存じの通り、"to post 500 dollars in revenue"
(収入として 500ドルを計上する)というふうに使われる他動詞であり、上記の posted
は、その直前の earned に対応していますね。earned の直後にまったく同じ
earned を繰り返すのは芸がないので、本質的には同じことを言いたいんだけど
posted というふうに言い換えているに過ぎません。

774 :名無しさん@英語勉強中:2015/12/15(火) 12:19:13.12 ID:5gGviDyS.net
>>770
>>772 で示したような combined という単語の語法については、「ウィズダム英和、第3版」
の combined の項目(combine ではなく、combined という単語の項目)にて、
形容詞として、次のように紹介しています。

   the size of France and Germany combined (フランスとドイツを合わせた大きさ)

775 :名無しさん@英語勉強中:2015/12/15(火) 12:23:12.46 ID:WPcPf00x.net
The age of modernity started around
the time of Machiavelli,the Italian materialist, and from his foundational political ideas came the main principles of modern thought.

and from〜からの文の主語って倒置なので
the main〜からで良いでしょうか?

776 :名無しさん@英語勉強中:2015/12/15(火) 12:25:35.44 ID:odG+bGrv.net
What is it to be human

この文のSVCなどを教えてください

777 :名無しさん@英語勉強中:2015/12/15(火) 12:35:32.08 ID:5gGviDyS.net
>>775
人にわかる日本語で書いてください。

778 :名無しさん@英語勉強中:2015/12/15(火) 12:37:49.95 ID:5gGviDyS.net
>>776
それが文として完結しているのか、あるいはタイプミス(文法ミス)を含んでいるのか、
そのままではわからないでしょう?もし文として完結しているのなら、ちゃんとピリオドか
question mark か何かを、原文通りにきちんと書いて下さい。

779 :名無しさん@英語勉強中:2015/12/15(火) 13:38:21.16 ID:WPcPf00x.net
>>778

すいません

小さいですがピリオドはあります

780 :名無しさん@英語勉強中:2015/12/15(火) 13:57:25.28 ID:5gGviDyS.net
>>779
What is it to be human.

上記のように、ピリオドがついていて、これが一文となっているわけですね。
それなら、これは間違った英文ということになります。
   What is it to be human?
これなら、「人間であるということは、どういうことなのか?」という疑問文になります。
もしもあなたの抱えている一文が、前後の文章の間に挟まっているのであれば、
その前後の文章を見れば、本当の意味がわかります。通常、あなたの例文の
ように it is という語順になっているものは、次のような文脈で使います。

   If the subject of ethics is ●what it is to be human●,
   and if what it is to be human is in part non-human,
   then the subject of ethics is not simply centred on humanity.
    (ネットから拾った文章)

781 :777 ◆TFWBMdHdF7zL :2015/12/15(火) 14:28:27.15 ID:YPT6i7O0.net
>>766
「that's sucks!」じゃないだろ。
That sucks! じゃないと意味が通らない。
>>764を読め。
辞書くらい引け。

782 :名無しさん@英語勉強中:2015/12/15(火) 14:38:02.70 ID:WPcPf00x.net
>>780


すいません>>775の文と間違えてました

>>775の英文構造 SVOCみたいなのを教えてほしいです

783 :名無しさん@英語勉強中:2015/12/15(火) 14:41:42.00 ID:MtkJGY93.net
>>780
すいません おっしゃる通りクエッションマークが抜けてました

What is it to be human?
この英文が正しいです

784 :えワ:2015/12/15(火) 15:27:09.97 ID:EVKrpXSx.net
>>782
普通に頭から訳せばいい。 SV and SVC

The age of modernity
started
around the time of Machiavelli,the Italian materialist,
and
from his foundational political ideas
came
the main principles of modern thought.

時代区分としての近代は、イタリアの唯物論者マキャベリの辺りから始まる。
彼の基本的な政治的なアイディアは、現代の思想の基礎になっている。

マキャベリ 生年月日: 1469年5月3日〜1527年6月21日
ガリレオ  生年月日: 1452年4月15日〜1519年5月2日
歴史の説明として、あまりいい英文ではない、多分、これでは×が付く。
英語としては、深入りしない方がいい。
あとは、「ルネッサンス」をググって見ておいた方がいいね。

785 :えワ:2015/12/15(火) 15:56:32.66 ID:EVKrpXSx.net
>>781
チンコピアス、いらっしゃい。
自演は、ほどほどにしてね。

辞書は色々あるからね。
WordNet 3.6
v) suck be: inadequate or objectionable "this sucks!"

Webster
Suck: To be objectionable, of very poor quality

786 :名無しさん@英語勉強中:2015/12/15(火) 16:44:49.68 ID:rEjeKVOM.net
彼は万事うまく行くと言い張ったが、私は心配で仕方がなかった

↑をおれは↓に訳したのだけれど不自然かな?
He insisted on everythig would be well done, but I was afraid that

本に載ってる訳例は↓
he insisted that everything would be all right, but I couldnt help feeling worried

787 :名無しさん@英語勉強中:2015/12/15(火) 17:35:02.70 ID:5gGviDyS.net
>>775
(1) The age of modernity
  近代という時代は、
(2) started around the time of Machiavelli, the Italian materialist,
  イタリアの唯物論者であるマキアヴェリの時代あたりに始まったが、
(3) and from his foundational political ideas
  彼の基礎的な政治思想から
(4) came the main principles of modern thought.
  近代思想の主な原理が生まれたのである。

★最後の (4) は、"V + S" という形になっている。
(3) と (4) を合わせた一文は、倒置の文。

788 :名無しさん@英語勉強中:2015/12/15(火) 17:37:59.14 ID:WPcPf00x.net
>>787

ありがとうございますm(__)m

789 :名無しさん@英語勉強中:2015/12/15(火) 17:39:26.33 ID:WEfYKz4Y.net
What is it to be human?

この文の文構造が良く分からないです

どれがSでどれがCなのでしょうか

790 :名無しさん@英語勉強中:2015/12/15(火) 18:02:04.14 ID:5gGviDyS.net
>>789
>>What is it to be human?

文型を云々するのが嫌いなんだけど、この文を理解するときに、まずは次の文を
考えてほしい。
  It is a good thing to be human. (= To be human is a good thing.)
    人間であるということは、いいことだ。
次に、この "a good thing" を what に置き換えて疑問文を作ると、
あなたの抱えている文になる。

791 :名無しさん@英語勉強中:2015/12/15(火) 18:45:13.42 ID:2A0lfWZF.net
よいというのは個々人の経験なので、aがつかないのはどうも気持ちが悪くないですか?

792 :名無しさん@英語勉強中:2015/12/15(火) 19:15:29.39 ID:tkH44F9T.net
この前、ちょっとしたサークルで、英語圏の人と喋ったんだけど
農学部って言うから、どんなことをしているの?って英語で
what do you doing ? って言ったら、ちょっと、ぽかんとされたのですが
これって通じる言葉ですか?

793 :えワ:2015/12/15(火) 19:28:43.91 ID:EVKrpXSx.net
>>789
>What is it to be human?
it = to be human
文型は What is it ? と同じでSVC。

What is to-be-human ?
It is "to be human."
「人間である」というのはどういう事か?
それが「人間である」という事。


>>792
せめてこれぐらいは、という感じかな。
What do you study ?
What are you studying ?

What do you studying ? なら文法としては「変」だけど通じるかな。

794 :名無しさん@英語勉強中:2015/12/15(火) 19:54:30.38 ID:A7SZQcsa.net
I often feel a stronger affinity with historians than I do with film studies folks.

than I do with film sudies folks という部分がよくわからないですが。
それに、thanの後ろは何なのでしょうか?

795 :名無しさん@英語勉強中:2015/12/15(火) 20:44:34.44 ID:tkH44F9T.net
793です
エワさん。サンクスです。ひひ
studyと言うと、勉強をしているという意味ですが、
私的には、農学部で、どんな事をしているのですか?
例えば、土を扱っているとか、という事を聞きたかったのですが
日本人同士の会話と同じみたいに、ツーカー的に通じる為には、
まだ、私の言葉不足なのかもしれませんねー
頑張りマッス!

796 :名無しさん@英語勉強中:2015/12/15(火) 20:45:34.36 ID:tkH44F9T.net
ひひは、消してくださいー

797 :名無しさん@英語勉強中:2015/12/15(火) 20:59:45.10 ID:5gGviDyS.net
>>794
>>I often feel a stronger affinity with historians
than I do with film studies folks.

than のあとは、しいて言えば次のように書ける。
  than the affinity that I feel with film studies folks.

これはもちろん、I run faster than Ted does. と同じ理屈で do を使ったもの。
film studies folks は、folks (= people) who do film studies
つまり students (and professors) of film studies、言い換えれば
people majoring in film studies という意味。

798 :名無しさん@英語勉強中:2015/12/15(火) 21:21:04.01 ID:DYfP++Q1.net
You people は差別的なので使わない方がいいって本当ですか?

799 :名無しさん@英語勉強中:2015/12/15(火) 21:34:21.00 ID:gVlIVwp+.net
>>772
ありがとう

800 :名無しさん@英語勉強中:2015/12/15(火) 22:24:50.03 ID:fhWd7MJB.net
>>789
what=O
is=V
it=S
to be human=C
でしょうか。すみません、当方、文法音痴です

801 :名無しさん@英語勉強中:2015/12/15(火) 22:41:58.35 ID:fhWd7MJB.net
isを挟んでS,Oはあり得ませんか
すみません、>800は忘れてください

802 :名無しさん@英語勉強中:2015/12/15(火) 23:44:40.89 ID:SPrean3z.net
He is easy to fool. (彼は騙しやすい)という記載が参考書にありました。
「騙す」の意味のfoolは他動詞のはずなのに目的語がないのは変な感じがします。
また、「騙しやすい」=「騙されやすい」という意味なので、to be fooledの方が
正しい表現ではないでしょうか。

803 :名無しさん@英語勉強中:2015/12/15(火) 23:50:49.00 ID:h6UphJj4.net
>>802
tough構文

804 :名無しさん@英語勉強中:2015/12/16(水) 01:20:44.84 ID:DbH1+IRU.net
オーガナ、オゥガナ ガナはgonnaですよね
オー、オゥって聞こえてる部分のspellを教えてくろさい!
バカですみません。

805 :えワ:2015/12/16(水) 01:48:25.41 ID:ui8+dCHY.net
>>804
all か Oh だろうね。
何の曲?

806 :名無しさん@英語勉強中:2015/12/16(水) 05:43:04.45 ID:wvz5zJo7.net
不規則動詞の活用形を原形過去形過去分詞…とaからzまでひたすら言っている教材をご存知の方いらっしゃいませんか?
子供の頃それを使っていて学生時代にすごく重宝したので、再勉強したい今すごく欲しくて…
全く同じものでなく似たものでも。

807 :名無しさん@英語勉強中:2015/12/16(水) 05:50:57.89 ID:GyVn76zJ.net
>>805allすね あ
りがとうございます!

808 :名無しさん@英語勉強中:2015/12/16(水) 06:16:52.08 ID:K+tDrGYn.net
>>806
Learn English ESL Irregular Verbs Grammar Rap Song
  Stick Stuck Stuck with Fluency MC!
    https://www.youtube.com/watch?v=gNaDvAYC0Jw

ともかく、YouTube などで irregular verbs とか song というキーワードを使って
検索してみたらどうかな?

809 :名無しさん@英語勉強中:2015/12/16(水) 06:59:13.92 ID:/K2SB3Eu.net
This is a novel-like experience that does not hold your hand.
これはあなたの手を保持していない小説のような経験です。

上の英文をGoogle先生に訳してもらうと少々意味不明なのですが、意味が通るように訳すと、

『これはあなたを突き放す小説のような体験です。』

といった感じでいいのでしょうか?

810 :名無しさん@英語勉強中:2015/12/16(水) 07:56:01.97 ID:z+aYX9Jb.net
>>803
tough構文が成立する条件はなんですか?
例えばこういう文は聞かないのはなぜ?
He is likely to like. 「彼は(みんなに)好かれやすい」

811 :名無しさん@英語勉強中:2015/12/16(水) 09:41:05.10 ID:K+tDrGYn.net
>>798
you people は、確かに差別的に聞こえて、使わない方が無難なようです。
それについては、2年3か月前にこのスレで長々と議論されました。

   http://yomogi.2ch.net/test/read.cgi/english/1377007445/618-679

上記のスレの 618 から 679 番までのレスを読んでみればわかります。なお、
678 番のレスにおいては、ネット上で拾ったネイティブたちによると思われる
"you people" という言葉についてのニュアンスについての説明が載っています。

なお、652 番から 655 番までのレスにおいては、you people を含んだ
OED から拾った20個の例文のうち、少なくとも 10 例は悪い意味で使っている
ことが明らかだということを述べています。

812 :名無しさん@英語勉強中:2015/12/16(水) 09:52:42.73 ID:K+tDrGYn.net
>>811
さっきは少し不正確なことを言ってしまいました。you people は、乱暴な言い回しに
聞こえることがあり、さらには差別的な言い回しに聞こえる ●こともある● ということです。
常にそうなのかどうかは、定かではありません。しかし、乱暴に聞こえる時が少しでも
あるような言い回しは、僕ら外国人は最初から使わないようにした方が無難だということです。

813 :名無しさん@英語勉強中:2015/12/16(水) 10:02:38.83 ID:K+tDrGYn.net
>>810
"AAA is easy to like." なら、使われていますね。

(1) I like Ike and Ike is ●easy to like●.
(2) But the ensemble cast, especially Harris, is ●easy to like●.
   (Corpus of Contemporary American English より)

814 :名無しさん@英語勉強中:2015/12/16(水) 10:53:06.09 ID:EXvNK7s4.net
ネイティブの表現をみてると英単語の途中を適当に大文字にしてるのをみかけます
どういうニュアンスなんでしょうか?

ActoR
ACtoR
AcToR

とか

815 :名無しさん@英語勉強中:2015/12/16(水) 12:01:34.99 ID:K+tDrGYn.net
>>814
真面目な回答がほしいのなら、その前後をきちんと引用しないとダメですよ。
それぞれの綴り方が、どんな文脈で出てきたかによって、説明の仕方、
解釈の仕方がまるで違ってきます。

816 :名無しさん@英語勉強中:2015/12/16(水) 12:22:28.72 ID:CXQdE2Bw.net
>>810
手元の参考書だと「難易を表す形容詞」が使われている場合にtough構文が成立するようです。

817 :名無しさん@英語勉強中:2015/12/16(水) 13:16:34.83 ID:XDjOg9wu.net
I suppose it was a way of showing off, and I suppose it worked because I was
the guy you went to if you wanted to know how to use a particular feature.
この文を自分で翻訳したら、

私はこれは自慢する方法だったのだとおもう、私はそれが役に立ったのは次の理由だったからなのだと思う。私は
もしあなたがこの機能がどう動くのか知りたいと思ったらあなたの頼りになる男だったという理由だ。

という訳にだと思うんですよ。なぜならgo-to guyが頼りになる男という意味だからです
でも著作権の関係上題名と翻訳文は言えませんが、この本の日本語訳をamazonで検索して中身検索をみたら
全く別なことが書いてありました。どうしてその役になるのかわかりません教えてください

818 :名無しさん@英語勉強中:2015/12/16(水) 13:36:44.04 ID:g+n4q8xK.net
引用はいいだろ
その別の訳を書き抜けよ
めんどくさいなあ

819 :名無しさん@英語勉強中:2015/12/16(水) 13:59:54.83 ID:GC6eNalC.net
ニックネームとかを-や_で区切ってるのをみかけるけど、
英語圏では-と_でどういうニュアンスの違いがある?

ying_masa とか teru-4ever

820 :名無しさん@英語勉強中:2015/12/16(水) 14:21:41.98 ID:g+n4q8xK.net
アンダースコアは純粋にスペースの代わりだな
スペース使えない時あるから
ハイフォンはそれプラス
普通に考えれば複合語とも読めるだろ

821 :名無しさん@英語勉強中:2015/12/16(水) 15:01:53.32 ID:NcZCrKJ2.net
>>817
その work は、「役に立った」じゃないでしょうに

822 :えワ:2015/12/16(水) 15:21:43.45 ID:ui8+dCHY.net
>>817
自分の英語力に見合った英文を読もう。
今の君の英語力で読める記事ではない。
眺めてるだけ、時間のムダだよ。

前文がなければ、feature の意味が取れない。
When I was a young journeyman programmer,
I would learn about every feature of the languages I was using,
and I would attempt to use all of those features when I wrote.

後文のこっちがこの記事の本旨
Most programming languages contain good parts and bad parts.
I discovered that I could be a better programmer by using only the good parts
and avoiding the bad parts. After all, how can you build something good out of bad parts ?

823 :名無しさん@英語勉強中:2015/12/16(水) 15:52:18.84 ID:XDjOg9wu.net
>>817
じゃあ、なんですか?

>>822
ごめん、何言ってるのか分からないです。

824 :名無しさん@英語勉強中:2015/12/16(水) 16:01:46.52 ID:XDjOg9wu.net
>>821
じゃあ、なんですか?

825 :名無しさん@英語勉強中:2015/12/16(水) 16:17:12.05 ID:XDjOg9wu.net
>>818
著作権法で保護されていますと書いてあるけれど書きますよ
引用はありということで。
「私@が@(@あ@な@た@も@お@そ@ら@く@か@つ@て@そ@う@で@あ@っ@た@よ#う#に#)#そ#れ&ぞ&れ&の&機&能&の&使&い&方&を!知!り#た$い%と$思#っ"ていたからだとおもう。」
一行飛ばしに読んでください、検索に引っかかって逮捕されたくないので

826 :えワ:2015/12/16(水) 16:28:27.01 ID:ui8+dCHY.net
>>825
お前、馬鹿だろう。
引用をするなら、ちゃんと引用しよう。最低、著作者の名前があれば、検索は出来るから。
どうやら、引用とパクリの区別が出来ない人のようだね。
>「一行飛ばしに読んでください、」← 無理だね、日本語が、「とっても不自由」かな。
日本語学校に通った方がいいと思う。

Cleverness, Douglas Cockroft said
"When I was a young journeyman programmer, I would learn about every feature of the languages I was using,
and I would attempt to use all of those features when I wrote.
<質問文>
I suppose it was a way of showing off, and I suppose it worked because I was
the guy you went to if you wanted to know how to use a particular feature.

Eventually I figured out that some of those features were more trouble than they were worth. "

827 :名無しさん@英語勉強中:2015/12/16(水) 17:16:00.69 ID:ACnElS4E.net
We have reached new culinary lows.

これって新たな食の低みに達したってことですよね?

828 :名無しさん@英語勉強中:2015/12/16(水) 18:34:43.92 ID:7zHpq24Q.net
「シェケナベイビー」だとZIPPO法に引っ掛かりそうだから、
「シェケナベイブ」にしたほうがよいと思わぬか各々

829 :名無しさん@英語勉強中:2015/12/16(水) 19:49:00.80 ID:XDjOg9wu.net
>>2
すみませんスルーするの忘れてました。以後気をつけます。

830 :名無しさん@英語勉強中:2015/12/16(水) 19:52:23.21 ID:K+tDrGYn.net
>>828
カタカナで書かれると、最初はさっぱりわからなかったが、あとになって気づいた。
Shake it now, baby! という意味だったのか。実に軽薄な感じのする言葉だな。

   http://www.lyricsfreak.com/j/junior+wells/shake+it+baby_20828096.html

その言葉が出てくるこんな歌もある。

831 :名無しさん@英語勉強中:2015/12/16(水) 20:08:20.70 ID:0M7Yaewh.net
>>811
ロングマン英和辞典   you people

1.((インフォーマル・話))(人に向かって)おまえら

What's matter you people?
おまえらどうしたんだよ?

2.((やや古))(呼びかけで)皆様

832 :名無しさん@英語勉強中:2015/12/16(水) 20:09:31.66 ID:0M7Yaewh.net
訂正
>>811
ロングマン英和辞典   you people

1.((インフォーマル・話))(人に向かって)おまえら

What's the matter with you people?
おまえらどうしたんだよ?

2.((やや古))(呼びかけで)皆様

833 :名無しさん@英語勉強中:2015/12/17(木) 08:07:12.05 ID:HYE0R3iD.net
すみません質問です。
医学部学士編入試験の問題ですが回答がないため文構造のご教授お願いします。
'' ''間を和訳せよという問題です。
ちなみにcessationは1級レベルなんですね。手持ちのパス単1級見たらありました、

A neoplasm, as defined by Willis, is ''an abnormal mass of tissue the growth of which exceeds and is uncoordinated with that of the normal tissues and persists in the same excessuve manner after the cessation of the stimuli which evoked the change.''

834 :名無しさん@英語勉強中:2015/12/17(木) 08:30:26.22 ID:77OPjbta.net
>>825 >>817に回答できる方のみお願いします。

835 :名無しさん@英語勉強中:2015/12/17(木) 08:32:47.80 ID:77OPjbta.net
>>700 >>753 >>755 回答できる方のみお願いします。

836 :えワ:2015/12/17(木) 09:07:23.25 ID:6FxsV5f1.net
>>833
英語ではなく、日本語の問題だね。

腫瘍(しゅよう、Neoplasm)Willis の定義
組織、細胞が生体内の制御に反して自律的に過剰に増殖することによってできる組織塊のこと。
病理学的には、新生物(しんせいぶつ、Tumor)と同義である。

837 :名無しさん@英語勉強中:2015/12/17(木) 09:07:23.68 ID:UN9h/Ec9.net
>>833
As defined by Willis(M)(Willisが定義するように)
A neoplasm is "an abnormal mass of tissue(SVC)
the growth of which(S)関
exceeds(V1) and
is coordinated with (V2)
that(=growthかmass) of the normal tissues (O)
and
persists (V3)
(in the same excessive manner)M (after the cessation of the stimuli)M
which(S) evoked(V) the change(O)."関

838 :名無しさん@英語勉強中:2015/12/17(木) 09:08:49.55 ID:77OPjbta.net
>>2
>>★「えワ」に注意!スルー推奨!!★

839 :名無しさん@英語勉強中:2015/12/17(木) 09:28:00.53 ID:e5PWe1eg.net
>>811
解答ありがとうございました。
You guys がくだけた感じなので you people はより丁寧な言い方だと思っていました。
例えば旅行に行って、数人の前で

You people are all tall! We Japanese are short nation so I envy you.

みたいに褒めてるつもりでも、誤解を招きうるのでしょうね?
英語は面白いけど難しいです。

840 :名無しさん@英語勉強中:2015/12/17(木) 11:01:54.50 ID:V0AjDyOg.net
He also criticized the 20 American historians for the nature of their reaction: Instead of responding to the Japanese government’s call to correct the information, they took it to task for trying to do so

このsoって何を指してるんでしょうか?

http://www.japantimes.co.jp/news/2015/12/11/national/history/fifty-japanese-scholars-attack-mcgraw-hill-u-s-academics-on-comfort-women-issue/

841 :名無しさん@英語勉強中:2015/12/17(木) 11:03:19.65 ID:Y5QZtbpb.net
All visitors to the US should line up at booths one through 15.

入国者は15番のブースにならべって言ってるのはわかるんだけど
oneってのが何のために入ってるのかがわからない

なんで?

842 :840:2015/12/17(木) 11:05:37.30 ID:V0AjDyOg.net
あとtook it to taskのitは何を指してるんでしょうか?

843 :名無しさん@英語勉強中:2015/12/17(木) 11:08:37.13 ID:Lkgbls7y.net
>>839
ネイティブはそういう知ったか的なへんてこ英語が一番きついって言ってたよw
普通に喋れよってw

844 :名無しさん@英語勉強中:2015/12/17(木) 11:14:00.66 ID:VcN1gP6o.net
>>841
booths one through 15 = from booths 1 to 15 = ブース1から15まで(ブース1からブース15まですべて)

このような through が to と同じ意味を持つアメリカ英語であることについては、
辞書を見ればわかる。

   Exits 1 through 5 (出口1から出口5まで)
   pages 1 through 654 (1ページから654ページまで)

845 :名無しさん@英語勉強中:2015/12/17(木) 11:19:32.62 ID:nEOsH57d.net
>>841
英語では1〜10までは単語で書き、後はアラビア数字を使うそうです。

846 :777 ◆TFWBMdHdF7zL :2015/12/17(木) 11:22:34.19 ID:qiGf+7Jy.net
>>840
take someone to task は someone を非難するという意味のイディオム。
http://idioms.thefreedictionary.com/take+to+task

took it to task の it は the Japanese government を指している。

trying to do so の so は to correct the information を指している。

847 :名無しさん@英語勉強中:2015/12/17(木) 11:23:18.27 ID:Y5QZtbpb.net
>>844
>>845
なるほど

今、翻訳サイトで翻訳したら

米国へのすべての訪問者はブース15を介して、一列に並ぶ必要があります。

って出たから、ブース15に一列に並べって意味だと感じ始めてた。
翻訳サイトは間違いでブース1〜15のいずれかに並べって意味なのか。

848 :名無しさん@英語勉強中:2015/12/17(木) 11:28:07.43 ID:VcN1gP6o.net
>>840
they took it to task for trying to do so

  = the 20 American historians took the Japanese government to task
   for trying to correct the information.

849 :名無しさん@英語勉強中:2015/12/17(木) 11:30:11.98 ID:VcN1gP6o.net
>>848 は、777 による >>846 のコメントとかぶってしまった。すまん。

850 :名無しさん@英語勉強中:2015/12/17(木) 11:59:01.94 ID:JSWrXAhV.net
>>839
自虐ジョークには聞こえるが、racism 的なものを感じるからよくない。

851 :名無しさん@英語勉強中:2015/12/17(木) 14:03:27.54 ID:I+ix17aK.net
>>836
>>837
ありがとうございます。
スッキリしました。
しかし結構複雑な文ですね…

852 :名無しさん@英語勉強中:2015/12/17(木) 14:06:12.78 ID:Lkgbls7y.net
どこがだよw

853 :名無しさん@英語勉強中:2015/12/17(木) 15:43:01.65 ID:O2b+xsFh.net
When all the statistical effects are taken into consideration,
Dr. Cranmer said, the bump in the Atlas data had about a 1-in-93 chance of being a fluke — far stronger than the 1-in-3.5-million odds of mere chance,
known as five-sigma, considered the gold standard for a discovery

consideredの主語がと役割がわかりません

854 :名無しさん@英語勉強中:2015/12/17(木) 15:56:14.61 ID:VcN1gP6o.net
>>853
considered の前に being または which is を付け加えて解釈すればいい。
それが修飾している(というか、それが指し示している)名詞は、
たぶんその前の the 1-in-3.5-million odds of mere chance
だろうと思うけど、確信はない。そのずっと前の名詞なのかもしれない。

855 :名無しさん@英語勉強中:2015/12/17(木) 15:57:45.43 ID:77OPjbta.net
>>825 >>817に回答できる方のみお願いします

856 :622:2015/12/17(木) 17:24:08.51 ID:JF8T9xu8.net
>630
ありがとうございました。
お礼が遅れてすみません。

857 :名無しさん@英語勉強中:2015/12/17(木) 17:27:59.19 ID:VcN1gP6o.net
>>853
considered の前に being または which is を付け加えて解釈すればいい。
それが修飾している(というか、それが指し示している)名詞は、
たぶんその前の the 1-in-3.5-million odds of mere chance
だろうと思うけど、確信はない。そのずっと前の名詞なのかもしれない。

858 :名無しさん@英語勉強中:2015/12/17(木) 17:40:22.09 ID:VcN1gP6o.net
>>853
the 1-in-3.5-million odds of mere chance,
known as five-sigma,
[which is] considered [to be] the gold standard for a discovery

あれから少しだけ考え直してみたけど、やはり上記のような語句を補って考えれば
いいと思う。ただし僕は理科が苦手なので、この種の問題にあまりついては
いけないので、文脈から外れて解釈していたら申し訳ない。

さらに上記の considered が修飾している名詞は、さっきも言ったように
the 1-in-3.5-million odds of mere chance だ。

859 :えワ:2015/12/17(木) 20:54:19.26 ID:6FxsV5f1.net
>>853
出典:ニューヨークタイムズ
タイトル:Physicists in Europe Find Tantalizing Hints of a Mysterious New Particle
ヒッグス粒子を確認!

five-sigma, considered the gold standard for a discovery
「発見」の黄金律と考えられている5シグマ
considered 以下は five-sigma にかかる形容詞句。

偏差値が分からなければ、理解出来ないだろうね。
英語の話ではない。

860 :名無しさん@英語勉強中:2015/12/17(木) 21:47:44.53 ID:KVlKg4nx.net
>>825
キチガイとしか思えない。英語の勉強をしている暇があったら精神病院に行け

861 :名無しさん@英語勉強中:2015/12/17(木) 22:09:15.82 ID:77OPjbta.net
答えられないなら黙っててもらえませんか?

862 :名無しさん@英語勉強中:2015/12/17(木) 23:49:43.83 ID:KVlKg4nx.net
著作権法で保護されていますと書いてあるけれど書きますよ
引用はありということで。
「私@が@(@あ@な@た@も@お@そ@ら@く@か@つ@て@そ@う@で@あ@っ@た@よ#う#に#)#そ#れ&ぞ&れ&の&機&能&の&使&い&方&を!知!り#た$い%と$思#っ"ていたからだとおもう。」
一行飛ばしに読んでください、検索に引っかかって逮捕されたくないので

これをキチガイと言わずして何と言おうか。The king of キチガイ

863 :名無しさん@英語勉強中:2015/12/18(金) 00:06:11.29 ID:4pLoSF+h.net
uh in terms of the track wide, I think Monaco is probably similar in a sometimes bent. But Monaco is such a smooth surface
bacause, uh, deep there now you know for there long time I knew day street and the re-surface everything a rear.
So when you arrive the Grand-prix a couple month ago, they are the everything dicking out and lay out fresh asfault every year
so I think Monaco is bumpiness surface such as smooth as Barbar of Indy Grand-prix very smooth, and you can see some undulation
but it is in Zero bumpy in Monaco and but here I mean propably have different you can see the manhole there is concrete pats one
corners debris 2-3 different some changes. And HS some difficult, but difficult means its great challengings for driver and if you
didn't have a confidense in such a tough 90 minutes race but I hope we can enjoying a *** race certainly for the fans *** from point
of view and it is great bacause, you know, I think it rollar action, rollar action I think they *** great job to modefy the circuit
and long straight and heavy breaking turn 3, so made it lot of opportunity overtakings and well as back street so I think ***
race to enjoy.

864 :863:2015/12/18(金) 00:10:19.74 ID:4pLoSF+h.net
https://www.youtube.com/watch?v=6NqivsxxUl0

たぶん100回以上の回数で、0430〜0550をディクテーションしてみました。
訂正や***の補充などをお願いします。

865 :名無しさん@英語勉強中:2015/12/18(金) 07:20:58.42 ID:Vzawl/WE.net
>>825 >>817 に答えられる人は一人もいないのですか?

866 :名無しさん@英語勉強中:2015/12/18(金) 09:34:16.35 ID:pC+ALBoH.net
質問です。あまりきちんとまとめられていないのですが。

「もし〜だったら/していたら、(今頃)○○していたのに」の
○○の部分が進行形というか、普通の文なら進行形で表されるような
(たとえば言っているのが11時として、10時半頃から今、その先も山を登っている状態とか)
文を英語で書く時の文法的な形、形の名前、キーワードやヒントなどをお願いします。
習慣的に山を登っている、ということではなくて、今日今この時に○○している、という感じで。
仮定法のなにかだとは思うのですが…。

867 :名無しさん@英語勉強中:2015/12/18(金) 10:04:52.34 ID:otAQ3GVR.net
>>866
たぶん、次のような言い回しでよいと思うんだけど、どうかな?

(1) If I ●had been studying● hard enough (then), I would have been
admitted to my favorite university by now.

(2) If I ●had been in the process of● working on my painting (then),
I would have finished it by now.

(3) If the project ●had still been going on● then, we would be
working on the next one now.

868 :名無しさん@英語勉強中:2015/12/18(金) 10:36:38.66 ID:14RMoGlb.net
普通に仮定法でええやん

869 :名無しさん@英語勉強中:2015/12/18(金) 10:56:27.64 ID:vrw+6GZY.net
(I supposer)It was a way of showing off, and (I suppose) it works ということは論理的に解釈すると
showing off がworksになる。 なぜなら was は linking 動詞だからね。
これを質問者の>>817 の訳に当て嵌めてみると「自慢が役立った」というおかしな結びつきになる。
どのような訳が適切なのかは質問文だけからだと分からないけど、少なくとも
両者のコロケイションを考慮して訳す必要があると思った。
単なる当て推量だけど「(何かを)誇示するとか、見せびらかして(show off) それがうまくいった」という
意味なんじゃないの。
それから went to に「頼りになる」とか凝った訳を当てているけど これも疑問だな。
you wen to は I(私)からすると came to だから 単に「やって来た」でしょ。

870 :名無しさん@英語勉強中:2015/12/18(金) 11:12:58.99 ID:9VG4UeD9.net
英語力が高1レベルなんだけど
受験英語じゃなくて旅行や日常会話の勉強をするには
どういう計画で行けばよいのかな?
効率よく手順を決めて進めていきたい。

単語熟語とか文法とかこれは使えるから先にやっとけとか
やり方でもお勧めの本でもなんでも

871 :名無しさん@英語勉強中:2015/12/18(金) 11:15:38.42 ID:JmjXpjWL.net
Under what conditions they became involved, in whose name and for what purpose,
and how they disappeared are issues that continue to hold deep significance
precisely because we need to learn from this history in order to stem its recurrence today and in the future

Under what conditions they became involved, in whose name and for what purpose,
and how they disappeared

how they disappearedが主語になるのはわかるんですが
Under what conditions they became involvedは前置詞から始まってるから副詞節なので主語になれないのではないでしょうか?
あとin whose name and for what purposeも前置詞から始まってるので主語になれないのではないでしょうか?

解説お願いします 👀
Rock54: Caution(BBR-MD5:0be15ced7fbdb9fdb4d0ce1929c1b82f)


872 :名無しさん@英語勉強中:2015/12/18(金) 11:16:55.81 ID:14RMoGlb.net
>>870
あのな、受験英語とか言っちゃってるけど高校でやるような内容は必須なんだよ
それ+表現や語彙を増やしていくだけなの。だから高1レベルとか言ってるなら早く高校卒業レベルまで勉強しろ。

873 :名無しさん@英語勉強中:2015/12/18(金) 11:18:30.50 ID:YGzbOpQH.net
効率よくいくなら、こいつの言う受験英語は避けて通れないなw

874 :名無しさん@英語勉強中:2015/12/18(金) 11:21:57.91 ID:vpfmSs6x.net
英会話はどんな方法でも会話する機会を
継続的に作らないといけない
最短や効率はその経験の中から考えるもの
金や暇によって英会話の機会の作りかたは
異なるが基本はそういうこと

ただ、独習法としては瞬間英作文が万人向け

875 :名無しさん@英語勉強中:2015/12/18(金) 11:48:51.69 ID:9VG4UeD9.net
>>874
ありがとう。
瞬間英作文か。調べてみる。

876 :名無しさん@英語勉強中:2015/12/18(金) 11:52:34.41 ID:JfX0xkva.net
理系論文を読んでいて分からないので質問します。
In addition, 4 and 3 of the 5 β-lactamases in A.xylosoxydans did not have any hits in E.coli.

この文なんですが4 and 3はどう訳せばいいでしょうか?
5つのβラクタマーゼのうちの4つと3つですか?
お願いします。

877 :名無しさん@英語勉強中:2015/12/18(金) 11:57:49.54 ID:eYjIGWWI.net
>>871
Under what conditions they became involved, (S1)名詞節
「どの様な状況で巻き込まれたか」
in whose name and for what purpose, and how they disappeared (S2) 名詞節
「誰の名義で、何のために、そしてどのように彼らが消えたか」
are (V) issues (C)「がissuesだ」

878 :名無しさん@英語勉強中:2015/12/18(金) 11:59:56.59 ID:JfX0xkva.net
あ、すみません解決しました。
最後にE.coli and P.aeruginosa.respectively.とあるんでそれぞれ3つと4つのβラクタマーゼですね。
失礼しました。

879 :名無しさん@英語勉強中:2015/12/18(金) 12:00:35.10 ID:Vzawl/WE.net
>>869 ありがとございます。>>825にあるプロの翻訳者が翻訳し文は滅茶苦茶ですね。
正解は
それは私が自慢する方法であった、そしてその自慢する方法はうまくいった。なぜなら。私は
あなたがやってくる男だったからだ。もし〜だったら。
ですよね。
このyouは漠然とした人を指すyouですよね。

880 :名無しさん@英語勉強中:2015/12/18(金) 12:03:38.02 ID:2Mm3t3Kr.net
>>854
>>857
ありがとう

881 :名無しさん@英語勉強中:2015/12/18(金) 12:37:28.16 ID:wqRPFzzm.net
英太郎は全部、ネット上で拾うか参考書から拾うので本当に英語がわかってるのか?と思うときがある。

ネットや参考書が間違ってればすべて間違えるし、あってれば言ってることはとりあえず正しいという感じ。

882 :名無しさん@英語勉強中:2015/12/18(金) 12:41:42.05 ID:14RMoGlb.net
英太郎は精度のよくない自動検索AIだと思っとけ

883 :名無しさん@英語勉強中:2015/12/18(金) 12:43:06.76 ID:14RMoGlb.net
英太郎は精度のよくない自動検索AIだと思っとけ

884 :名無しさん@英語勉強中:2015/12/18(金) 13:07:25.40 ID:fgy0A/96.net
英太郎は精度のよくない自動検索AIだと思っとけ

885 :名無しさん@英語勉強中:2015/12/18(金) 13:46:05.28 ID:pC+ALBoH.net
>>866です。>>867, >>868, ありがとうございます。
仮定法や完了形についてきちんと理解できていないために混乱したらしいと分かりました。

復習しましたが、正直全然自信がないので、例文を考えて英訳してみました。
間違っている点についてご指摘お願いします。

もし8時に起きていたら、今頃山を登っている(最中の)はずだったのに。
If I have gotten up at eight, I would be climbing the mountain now.

886 :名無しさん@英語勉強中:2015/12/18(金) 13:50:06.40 ID:14RMoGlb.net
仮定法になってない

887 :名無しさん@英語勉強中:2015/12/18(金) 14:00:55.98 ID:pC+ALBoH.net
>>885です。
すみません、
前半を完了形にしたのが間違いで
If I got up at eight, I would be climbing the mountain.
でしょうか?

888 :名無しさん@英語勉強中:2015/12/18(金) 14:05:22.94 ID:14RMoGlb.net
違う

889 :名無しさん@英語勉強中:2015/12/18(金) 14:35:02.45 ID:e1mHDPUA.net
That is the sort of question one would expect to hear from a 排外主義者 .

これの構文を教えてください
関係代名詞だと思うんですがone以降は完全文ですよね?

890 :名無しさん@英語勉強中:2015/12/18(金) 14:37:22.88 ID:14RMoGlb.net
構文も何も中学レベルじゃねえか

891 :名無しさん@英語勉強中:2015/12/18(金) 14:42:14.88 ID:vpfmSs6x.net
one would expect to hear the sort of question from a 排外主義者
と読めよ
ビジュアル英文解釈からやり直せ

892 :名無しさん@英語勉強中:2015/12/18(金) 14:53:49.08 ID:vpfmSs6x.net
このレベルの構文取れないで速読英単上級というのはアンバランスやな
英検2級レベルの読み込みが足りてない

893 :名無しさん@英語勉強中:2015/12/18(金) 15:43:45.41 ID:WPQq/PsK.net
TOEICで800目指したほうがいいかも

894 :名無しさん@英語勉強中:2015/12/19(土) 01:17:23.83 ID:mylbQNrq.net
Will these materials and these historians be declared ‘state secrets’ under Abe administration-related efforts such as this letter?
Will it become possible in today’s Japan to declare that this history did not happen, when many of us in Japan and around the world possess historical materials that prove it took place?”

declaredの目的語がstate secretsでしょうか?なんで declared as state secretsではないんでしょうか?

prove it took place のitはthis historyでいいでしょうか?
あと起こったと証明するのに この歴史は起こらなかったと宣言すると書いてありますが、意味的に矛盾するのではないでしょうか?

895 :名無しさん@英語勉強中:2015/12/19(土) 01:29:23.51 ID:TvvPuuNi.net
>>894
declearは第5文型 目的語はthese materials and these historians
passiveじゃない文を自分で書いてみな

実際に起こったという証拠を持ってる人がいるのに、「それは起こらなかった」と言い張るとか頭おかしくね?
っていう話なので何も矛盾しとらんよ

896 :名無しさん@英語勉強中:2015/12/19(土) 01:32:04.69 ID:TvvPuuNi.net
ああ あとitはthis historyであっとるね

897 :名無しさん@英語勉強中:2015/12/19(土) 05:12:44.62 ID:QpgSaIoc.net
I arrived one hour late
I arrived an hour late

どっちでも可だとおもうんですが、違いは何でしょうか?
本来 a は”1つ”という意味だけではないと思います。数字と合わさることで意味が合致するのでしょうか?

898 :えワ:2015/12/19(土) 06:02:10.25 ID:Vkbt82gq.net
>>897
「どっちでもいい。」と答えてから。
人によって違うと思うけど、私の使い分けはこんな感じ。

1時間の「意味の重さ(強さ)」が変わって、「1時間遅れた」と「1時間ぐらい遅れた(随分遅れた)」の違い。
あるいは、「1時間の正確さ」の違いで、時計と腹時計の違い、
one hour を使ったら、two hour, three hour が意識されていて、
an hour は他の情報、例えば「間に合った、間に合わなかった」が意識されている。

「意味の重さ(強さ)というのはこんな感じ。
-「1時間遅れたんで、最初の講義は受けられなかった。」← 「受けられなかった理由」なんで、「1時間遅れた事」は無視出来ない(意味が重い)。
-「1時間遅れたけど、なんとか試験には間に合った。」← 試験は受けられてるんで、遅れた時間はどうでもいい(意味が軽い、弱い)。

899 :名無しさん@英語勉強中:2015/12/19(土) 06:51:16.58 ID:KgsVPre0.net
>>897
an hourならhourが中心、one hourならoneが中心。
遅れた時間が2時間や23時間でなく「いち」時間だと際立たせたい時にoneになる。
だから、普通に使う場合はan hour。How many hoursと訊かれた答えとかならoneが分かりやすい。
遅れたこと自体の重みは一切関係ない。

900 :名無しさん@英語勉強中:2015/12/19(土) 06:55:46.23 ID:A0tJ4/kl.net
重さってイミフだな
さすが集ストされてるのに気がつかないだけあるな

901 :えワ:2015/12/19(土) 07:12:49.70 ID:Vkbt82gq.net
>>900
君に説明してるわけではないんだけど、どういう意味かな?
まぁ、日本語が不自由な人だろうね。
君が分かりやすい説明をすればいいのに、
まぁ、英語が不自由な人なんだろうね。

で、お前、馬鹿だろう。

902 :名無しさん@英語勉強中:2015/12/19(土) 07:55:10.70 ID:QpgSaIoc.net
えワはもうこなくなったんですね。

えワもどきさん、ありがとうね。

903 :名無しさん@英語勉強中:2015/12/19(土) 07:56:40.21 ID:QpgSaIoc.net
えワがこんな論理的な長文かけるわけないから、
えワの口調と、フレーズをところどころにいれてるだけだね。

チンコピアス倒すのかな

904 :えワ:2015/12/19(土) 08:04:52.89 ID:Vkbt82gq.net
>>902
777氏のコテを使って成りすましてる荒らしがいるからね。
本物の777氏なら、「馬鹿」とののしる前に、馬鹿にも分かるように説明してくれる。
コテではなく、書いてる中身で判断するのがいいよ。

因みに、「偽コテ」は、今、火だるまになってる。

905 :名無しさん@英語勉強中:2015/12/19(土) 08:29:48.71 ID:A0tJ4/kl.net
カルトがお前を狙ってる

906 :名無しさん@英語勉強中:2015/12/19(土) 08:36:51.67 ID:J+kcikyM.net
瞬間英作文って効果ある?なんか日本語から英語に訳すって非効率って聞いた。初心者だからわからないが

907 :えワ:2015/12/19(土) 08:44:12.75 ID:Vkbt82gq.net
>>905
「山より大きなイノシシは出ない」と言ってね、共産党より大きなカルトは出て来ない。
で、共産党は、表面上は、法を守らなきゃいけない。
「傘下」のしばき隊が捨てられた理由だ。

908 :名無しさん@英語勉強中:2015/12/19(土) 09:03:55.76 ID:A0tJ4/kl.net
家電の音かわった?

909 :あぼーん:あぼーん
あぼーん

910 :えワ:2015/12/19(土) 09:19:41.09 ID:Vkbt82gq.net
新潟日報の坂本秀樹の処分の甘さが、仕掛けを許しているんだよ。
彼は、個人情報をツィッターでさらしたからね。
「新潟日報を守る」と決めた以上、ここでは動けないと思うけど、どうだろう。

911 :名無しさん@英語勉強中:2015/12/19(土) 09:32:38.00 ID:A0tJ4/kl.net
コンセントから音がする

912 :名無しさん@英語勉強中:2015/12/19(土) 10:33:45.66 ID:hVwKxTR/.net
>>902, 903 荒らしにエサを与えないでください。

913 :えワ:2015/12/19(土) 10:35:02.01 ID:Vkbt82gq.net
>>911
日本共産党の志位さんが、盗聴器を仕掛けたのかも知れないね。
人を殺した宮本賢治が委員長になれるんだから、盗聴器ぐらい、屁でもないだろう。

914 :名無しさん@英語勉強中:2015/12/19(土) 10:43:20.81 ID:EH+jBWdq.net
>>895
ありがとう
these materials and these historians will be declared state secretsなので

these materials and these historiansは主語じゃないんですか?

915 :914:2015/12/19(土) 10:58:56.27 ID:EH+jBWdq.net
(they) declared these materials and these historians state secretsだから
these materials and these historiansは目的語ですね

ぼくが間違ってた

916 :名無しさん@英語勉強中:2015/12/19(土) 11:03:52.83 ID:mlOa54Rh.net
My brother studies history in his college.
(疑問文にして、Noで答える)

だれかこれについてこたえて

917 :ココア ◆5VAetm2E2AHL :2015/12/19(土) 11:24:09.45 ID:fOHdrWio.net
>>916
http://eigo-gakushu.com/eigo1/20.html
http://english.005net.com/yoten/ippan.php
を読んで理解してみよう

918 :名無しさん@英語勉強中:2015/12/19(土) 11:31:17.18 ID:mlOa54Rh.net
916ですが、

http://i.imgur.com/5C70x9C.jpg

これの(5)になります
(3)や(4)は They are を Are theyにするのだから、
これに習って My brother dosent〜だと思うのです
回答ではYour brotherになってる

919 :名無しさん@英語勉強中:2015/12/19(土) 11:32:09.85 ID:mlOa54Rh.net
あとレスの番号飛んでますね
>>917見れないです

920 :えワ:2015/12/19(土) 11:37:07.07 ID:Vkbt82gq.net
>>916
考え過ぎてるね、ただ、his college は変。
>My brother studies history in his college. (疑問文にして、Noで答える)
兄は大学で歴史を勉強してる。
<疑問文>
あなたのお兄さんは大学で歴史を勉強していますか。
Does your borother study history in his college ?
<No で答える。>
No, he doesn't で足りるけど、ちがうんだろうね。
No, my brother doesn't study history in his college ?

921 :名無しさん@英語勉強中:2015/12/19(土) 11:43:30.81 ID:mlOa54Rh.net
>>920
なるほど
なおこの問題の原文が>>918に載せておきました

922 :名無しさん@英語勉強中:2015/12/19(土) 11:45:56.67 ID:mlOa54Rh.net
まとめて>>916>>918について質問しなおします。


http://i.imgur.com/5C70x9C.jpg

これの(5)
My brother studies history in his college.
(疑問文にして、Noで答える)

という問題ですが、なぜ Does my brother studies history in his college.
では間違いなのですか?
正解は Does your brother studies history in his college. となっている

私の兄はー
私の兄はー?
という流れでなぜ間違いなのか、そして
私の兄はー
あなたの兄はー
なぜ「あなた」がでてきたのか

923 :名無しさん@英語勉強中:2015/12/19(土) 11:46:51.84 ID:mlOa54Rh.net
あと、No he dosent で答えてます(正解は)

924 :名無しさん@英語勉強中:2015/12/19(土) 11:48:08.71 ID:kyUOsI7x.net
日常的に色々と苦労してそうなやつだなw

925 :名無しさん@英語勉強中:2015/12/19(土) 12:16:49.85 ID:eu8IHOG6.net
形式的に疑問文に書き直す場合か、もしくは自分の兄が大学で何を学んでいるのか知らないような状況だと
" Does my brother...?" 答えとして間違っていないと思うけどね。
でも、そういう問題って普通は会話としてのロールプレイ方式で答えるのが一般的だから
your が正解になっているんだよ。

926 :名無しさん@英語勉強中:2015/12/19(土) 12:22:22.26 ID:eu8IHOG6.net
後、stuies は疑問文では原形のstudy になるし、くえっションマークを文尾にね

927 :名無しさん@英語勉強中:2015/12/19(土) 12:28:28.06 ID:kyUOsI7x.net
落ち着けよw

928 :名無しさん@英語勉強中:2015/12/19(土) 12:53:57.10 ID:1IxWTUy8.net
>>222
>>536

929 :名無しさん@英語勉強中:2015/12/19(土) 12:57:01.26 ID:1IxWTUy8.net
すまん928は、
>>906向けです。瞬間何とかは知らないので

930 :名無しさん@英語勉強中:2015/12/19(土) 15:41:42.60 ID:z+JMflW8.net
ふざけた話の途中で
what do you thinking?
what do you talking about?
とアメリカ人に言われました。
英語を日本で教えている人なので驚いたのですが、普通ですか?

931 :名無しさん@英語勉強中:2015/12/19(土) 16:18:26.12 ID:2Vsb0eyc.net
Every NaN shall compare unordered with everything, including itself.
普通に読んだら
NaNはunorderdと全ての物(NaNを含めて)と比べられなければならない。
文法的にはあっているけれど意味的には間違ってますよね。NaNは順序不定として全ての物と比べられなければいけない
が意味的に正解だと思うのですがそうするためにはどのように解釈したらいいですか?

932 :名無しさん@英語勉強中:2015/12/19(土) 17:10:44.26 ID:F1zwqJKm.net
>>930
普通ではない
明らかに怒っている

933 :名無しさん@英語勉強中:2015/12/19(土) 17:25:19.25 ID:b50hAlCq.net
冠詞の省略に関連して。

the first and the second boys
the first and second boys

これでいいでしょうか。

934 :名無しさん@英語勉強中:2015/12/19(土) 17:26:45.98 ID:i3THgRsP.net
Under what conditions they became involved, in whose name and for what purpose,
and how they disappeared are issues that continue to hold deep significance
precisely because we need to learn from this history in order to stem its recurrence today and in the future

Under what conditions they became involved, in whose name and for what purpose,
and how they disappeared

how they disappearedが主語になるのはわかるんですが
Under what conditions they became involvedは前置詞から始まってるから副詞節なので主語になれないのではないでしょうか?
あとin whose name and for what purposeも前置詞から始まってるので主語になれないのではないでしょうか?

解説お願いします 👀
Rock54: Caution(BBR-MD5:0be15ced7fbdb9fdb4d0ce1929c1b82f)


935 :名無しさん@英語勉強中:2015/12/19(土) 18:24:31.13 ID:AK5OqQrh.net
CVSなので issues are と読めば解決しね?

936 :名無しさん@英語勉強中:2015/12/19(土) 18:56:25.62 ID:pPAPmJfk.net
>>930
What are you thinking?
What are you talking about?
をうまく聞き取れなかっただけかと・・

what+areがくっついて発音されて ゥワダァのように聴こえる。
母音にはさまれたT などでググれば参考になるかも。

937 :名無しさん@英語勉強中:2015/12/19(土) 19:20:03.66 ID:AEsO0J6D.net
洋楽で勉強してるんですが、和訳がサイトによって全然違ってたり省略されてたりでよく分からず挫折しそうです。
洋楽でもっと効率的な勉強法はないですか?

938 :名無しさん@英語勉強中:2015/12/19(土) 19:25:25.96 ID:kyUOsI7x.net
そもそも洋楽で勉強というのは凄まじく効率の悪い英語学習法って知ってる?

939 :三年英太郎 ◆3CZBjOt3.Y :2015/12/19(土) 19:30:38.23 ID:jiWHQPCi.net
>>937
効率にこだわるなら、洋楽(の歌詞)で英語の勉強なんかしない方がいい。
少なくとも初級段階では。

両義性(あいまい)、省略、倒置、スラング、非文法 etc. が当たり前の世界だから。

昔、lady marmalade の歌詞をネイチブに質問したことがあるが、
ネイチブでさえ分からない箇所があった。

940 :名無しさん@英語勉強中:2015/12/19(土) 19:39:44.53 ID:eu8IHOG6.net
意味からするとare の前全部が主語のような気がするけどね。
「どのような状況で、誰の名で、どんな目的で彼らが関わり始め、どのように姿を消したかは
深い意味を持ち続けている問題だ」

941 :名無しさん@英語勉強中:2015/12/19(土) 19:50:21.16 ID:PsQG/Vfc.net
>>934
(一)前置詞句も主語になる
(ニ)前置詞付きの疑問詞は名詞相当の固まりをつくる
どっちかで考えればいいんじゃない?
(ニ)はknow to what extent SVとかknow at what time SVとか、
目的語になってるやつだと探しやすくてすぐ見つかる

942 :名無しさん@英語勉強中:2015/12/19(土) 20:14:12.65 ID:Hbfe4aBD.net
簡単な英文があるのですが、すんなり訳せそうでいて、どうもハッキリとした形に訳せません

You know what they say about all work and no play, after all.

多少大胆に意訳してでも、何とか日本語としてしっくりくる文に出来ないものでしょうか?
なお、この文は「カーナビについての説明」なので、それを踏まえた形で訳す必要があります。

943 :三年英太郎 ◆3CZBjOt3.Y :2015/12/19(土) 20:39:57.49 ID:jiWHQPCi.net
>>942
遊ばないがり勉君がどうなるって言われてるか、知ってるでしょ。

944 :三年英太郎 ◆3CZBjOt3.Y :2015/12/19(土) 20:44:29.35 ID:jiWHQPCi.net
カーナビの文章なら、play は両義的かもしれない。
普通の遊びって意味と、構造的な余裕という意味での遊び。

遊びがないギッチギチのものがどうなるって言われてるか、知ってるでしょ。

945 :名無しさん@英語勉強中:2015/12/19(土) 20:44:58.05 ID:Ymjx4VOi.net
>>942
何てったって、働いてばかりじゃ、ろくな人間になれないっていう言葉があるでしょうに。

946 :名無しさん@英語勉強中:2015/12/19(土) 21:57:50.95 ID:AEsO0J6D.net
>>939
やっぱりそうなんですか。。
洋楽だとモチベーション保てていいかと思ったんですがやっぱ非効率ですね

947 :名無しさん@英語勉強中:2015/12/19(土) 22:43:30.33 ID:PeEFeD6f.net
>>932
>>936
ありがとうございました。
ググってもあまり出てこないので、おかしいなと思ったのですが、確かにそんな気がしてきました。

948 :名無しさん@英語勉強中:2015/12/19(土) 23:17:35.37 ID:lww97ozs.net
海外のコミックを読んでいて、おそらく作中の人物が森に迷っているシーンでの台詞なのですが、上手く訳せません・・・。
Where th' hay are we?
という短い文章なのですが、th'が何の略かもわからず、hayも干草という意味しか出てこなくてth' hayの部分がまったく分かりません。
何卒ご教授願えないでしょうか。

949 :名無しさん@英語勉強中:2015/12/19(土) 23:26:39.32 ID:FQalarqO.net
Where the hell are we ?
かな
それが訛ったとか

950 :三年英太郎 ◆3CZBjOt3.Y :2015/12/19(土) 23:31:05.74 ID:jiWHQPCi.net
the hell を the hay とか濁すのですよ。
Oh my God! を ... Goodness とか、Jesus! を Gee! とかいうのと同じ。

th' = the

951 :三年英太郎 ◆3CZBjOt3.Y :2015/12/19(土) 23:33:12.35 ID:jiWHQPCi.net
fucking → freeking
shit → sugar

この手のものはあげればキリがない。

952 :三年英太郎 ◆3CZBjOt3.Y :2015/12/19(土) 23:33:37.77 ID:jiWHQPCi.net
*freaking

953 :名無しさん@英語勉強中:2015/12/19(土) 23:40:55.61 ID:lww97ozs.net
>>949-952
どうもありがとうございます!!
TH'何かの略かとは思ってたんですが、THEだったんですね…。
最近海外の漫画を読み始めたのですが、'boutとかダッシュを使った訛りの表現は多いのですね。
そしてhellがHayに変化するんですね…自力じゃ一生気づけそうにありませんでした…。
本当にありがとうございます!

954 :名無しさん@英語勉強中:2015/12/19(土) 23:47:54.82 ID:kyUOsI7x.net
変化じゃなくてそのまま言うとアレだから変えるんだよ
oh my godを goshにしたりな

955 :名無しさん@英語勉強中:2015/12/19(土) 23:49:58.66 ID:lww97ozs.net
ああなるほど、hellやgodとそのままだからアレだから、「濁す」ってことなのですね。
理解力不足でした、大変勉強になりましたありがとうございます。

956 :名無しさん@英語勉強中:2015/12/19(土) 23:50:56.91 ID:lww97ozs.net
×hellやgodとそのままだから
○hellやgodそのままだとアレだから
日本語力不足でした

957 :えワ:2015/12/20(日) 01:45:45.69 ID:sXwWVq/R.net
hell は、いわゆる four letter word ではないね。
また、騙されてるのかな。

958 :942:2015/12/20(日) 03:02:44.99 ID:7Mj2sGln.net
>>943-945
ありがとうございます。
うまい日本語は出てきませんが、言わんとしている事は大体分かったような気がします。

ちなみにこれは、ナビの機能や表示等を論評した文章なので、要するに意味する所は、
「最適化経路の表示を神経質にトレースする事だけが最適ではない」
といったあたりなのかな…

959 :名無しさん@英語勉強中:2015/12/20(日) 14:14:37.65 ID:DWpzNG3L.net
てす

960 :三年英太郎 ◆3CZBjOt3.Y :2015/12/20(日) 14:18:00.91 ID:vVDOqzJ9.net
>>958
これは All work and no play makes Jack a dull boy. ということわざを引用してるので、
文脈は分からないけど、遊びがないカーナビはつまらんというようなことでゎ?

961 :名無しさん@英語勉強中:2015/12/20(日) 14:22:26.76 ID:DWpzNG3L.net
>>937
具体的には何の曲ですかね

962 :名無しさん@英語勉強中:2015/12/20(日) 15:10:03.63 ID:i15C+G+R.net
>>961
ほぼワンオクです
洋楽ではないかもですが…

963 :名無しさん@英語勉強中:2015/12/20(日) 16:09:30.67 ID:5Z15hUvZ.net
・ by と near besaide の違いについて

by は接している。もしくは使っているなど関係が深い的な意味合いで「近い」(例えば、あなたに寄り添って(心情的に)、はby)
near は距離が近い
besaideは隣に

という感じで説明して大丈夫でしょうか?

964 :名無しさん@英語勉強中:2015/12/20(日) 16:14:45.30 ID:i4Kgeogf.net
>>931は質問が難しすぎて誰も答えられませんか?

965 :名無しさん@英語勉強中:2015/12/20(日) 16:24:37.63 ID:DWpzNG3L.net
>>962
うーむそれは確かに語学の役にたたないかも
(。>д<)

966 :名無しさん@英語勉強中:2015/12/20(日) 16:37:58.79 ID:DWpzNG3L.net
>>964どーでもいい内容なんで(。>д<)

967 :名無しさん@英語勉強中:2015/12/20(日) 17:41:19.93 ID:d1rt5V71.net
>>931
非数は、不正例外と非数を含むすべての数とを比較した結果生じる。
意味的には完全にあってる。

968 :名無しさん@英語勉強中:2015/12/20(日) 17:46:56.78 ID:d1rt5V71.net
非数と非数の比較でも非数が返されるってことだね。
演算時に少なくとも一方のオペランドが非数であれば、結果は非数が返されるからincluding itselfなんだよね。

969 :名無しさん@英語勉強中:2015/12/20(日) 17:47:49.57 ID:i4Kgeogf.net
>>967 unorderdというのは、以上、以下、未満、超過、比べられない、という分類のひとつなんですよ。「NaNとNaNを含む全ての数と比べると比べられないになる」という意味合いになるように解釈しなければいけないわけです。

970 :名無しさん@英語勉強中:2015/12/20(日) 17:51:46.45 ID:d1rt5V71.net
>>969
知ってるよ。なんで俺に言ってんの?

971 :名無しさん@英語勉強中:2015/12/20(日) 17:54:50.72 ID:d1rt5V71.net
つうかさ、自演釣りに乗ってやってるんだから、知らないわけないだろ。
それに、一般的な分類は大、小、等、不正だ。

972 :名無しさん@英語勉強中:2015/12/20(日) 17:57:19.41 ID:i4Kgeogf.net
あんまり言うと叩かれるので言いませんが、比較してもNaNは生じません。
生じるのはfalseかtrueのどちらかです。

973 :777 ◆TFWBMdHdF7zL :2015/12/20(日) 18:07:42.36 ID:KfjuM/56.net
60〜70年代の比較的スローな曲は英語学習に最適
この頃の歌詞はスラングも少なくわかりやすい。
メロディも美しいのが多い。

Where Have All the Flowers Gone - Kingston Trio
https://www.youtube.com/watch?v=4Qt57c7rnHM

Solitaire - The Carpenters
https://www.youtube.com/watch?v=Y0tUHbMnBNI

Moon River - Ann Margret
https://www.youtube.com/watch?v=PjZkic-VuNI

If Every Day Was Like Christmas Time- Elvis Presley
https://www.youtube.com/watch?v=1pRNMeIAqKk

Can't Help Falling In Love - Elvis Presley
https://www.youtube.com/watch?v=5V430M59Yn8

Puppet On A String - Elvis Presley
https://www.youtube.com/watch?v=Sm2s3-_OFWY

974 :名無しさん@英語勉強中:2015/12/20(日) 18:14:39.20 ID:WjKYLig6.net
>>965
そうか、
Bite a bullet が我慢するって意味なのは勉強になったけど別々で調べてたから意味わからなかったわ

975 :えワ:2015/12/20(日) 18:21:46.90 ID:9jFhaMw4.net
チンコピアスのつまらない自演が始まったか。

英語ではなく、「不同省小数点の演算」の規格の話。
訳せない、というより意味が取れないのは、その知識がないから。

<これは、NaN のウィキ>
NaN(Not a Number、非数、ナン)は、コンピュータにおいて、主に浮動小数点演算の結果として、
不正なオペランドを与えられたために生じた結果を表す値またはシンボルである。
NaNの体系的仕様は、1985年の IEEE 754 浮動小数点規格で標準が与えられている。

出典:What is the rationale for all comparisons returning false for IEEE754 NaN values?

<前のパラグラフ>
I mentioned that trichotomy does not hold for floating-point values.
However, there is a similar property that does hold. Clause 5.11, paragraph 2 of the 754-2008 standard:

< the 754-2008 standard: の規格>
Four mutually exclusive relations are possible: less than, equal, greater than, and unordered.
The last case arises when at least one operand is NaN.
Every NaN shall compare unordered with everything, including itself. ← で、これが質問文

976 :名無しさん@英語勉強中:2015/12/20(日) 18:24:27.67 ID:i4Kgeogf.net
>>2
★「えワ」に注意!スルー推奨!!★
 とくに、「えワ」に注意しましょう。珍回答の名手です。
 英語力が低いのですが、自信満々で回答するため、初めて訪れる人や初学者が騙される傾向にあります。
 「えワ」は、staffとstuffの区別も付かないレベルです。
・回答せずに、回答者・質問者を煽るだけの参加者もいます。いちいち煽られる必要はありません。
 あなたの目的は、疑問を解決することであり、煽りに口論で勝つことではないはずです。

977 :名無しさん@英語勉強中:2015/12/20(日) 18:35:30.43 ID:YjnvUFky.net
>>973
ちょっと古すぎだかも
いくらなんでも学校じゃないからな
(。>д<)

978 :名無しさん@英語勉強中:2015/12/20(日) 18:39:42.11 ID:M343dDnI.net
>>931
Every NaN shall compare unordered with everything, including itself.

compare with は「と比較する」じゃなく「に匹敵する、比肩する」の方で
「NaNは、自らを含め、順序不定ですべてのものに匹敵する(に相当する、になる)ものとする」

「すべてのもの」は上のless than, equal, grearer than,など。
じゃ意味が通じない?

979 :777 ◆TFWBMdHdF7zL :2015/12/20(日) 18:48:24.90 ID:KfjuM/56.net
>>977
>いくらなんでも学校じゃないからな

イミフ
この人は 20 代だが Carpenters とか Beatles も薦めている。

リスニングを強化したい人必見!英語学習に効く洋楽ソング10選
http://tabippo.net/english_songs/

980 :名無しさん@英語勉強中:2015/12/20(日) 18:48:33.66 ID:i4Kgeogf.net
>>978 違うとおもいます。いま思いついたんだけれどEvery Nan [that shall compre] unordered with everything, including itself.
ってことじゃないんですかね。

981 :えワ:2015/12/20(日) 18:52:52.52 ID:9jFhaMw4.net
>>980
つまらない自演は止めなよ。
チンコピアスが考えて、理解できる話じゃないから。

982 :777 ◆TFWBMdHdF7zL :2015/12/20(日) 19:00:58.05 ID:KfjuM/56.net
音楽というのは科学と違って、時代が新しいほどいい音楽が出てくるというのは嘘。
クラシック音楽を見ればわかる。
18、9 世紀のドイツ音楽がクラシック音楽の全盛時代で、それ以降衰退してる。
同様にポップスの全盛時代は 60 年代のアメリカ。

983 :名無しさん@英語勉強中:2015/12/20(日) 19:38:08.82 ID:UCZngfCm.net
ID:i4Kgeogfが何も分かってないことは分かるが、難しいね、これ。

984 :777 ◆TFWBMdHdF7zL :2015/12/20(日) 19:46:13.22 ID:KfjuM/56.net
>>983
お前が何も分かってないだけだろw

985 :777 ◆TFWBMdHdF7zL :2015/12/20(日) 20:01:29.61 ID:KfjuM/56.net
>>931
http://stackoverflow.com/questions/8627331/what-does-ordered-unordered-comparison-mean
An ordered comparison checks if neither operand is NaN.
Conversely, an unordered comparison checks if either operand is a NaN.

986 :名無しさん@英語勉強中:2015/12/20(日) 20:09:17.56 ID:QBH8VZTh.net
次スレ作ってみた

http://yomogi.2ch.net/test/read.cgi/english/1450609257/l50

987 :名無しさん@英語勉強中:2015/12/20(日) 20:13:16.70 ID:UCZngfCm.net
成る程、777ってコテハンも馬鹿だね。
根本的に分かってないことがよく分かる。

988 :名無しさん@英語勉強中:2015/12/20(日) 21:24:57.86 ID:CNDuc4Mq.net
無生物Sで考えられるのは、原因 手段 道具 以外で何かありますか?

989 :名無しさん@英語勉強中:2015/12/20(日) 22:32:20.70 ID:d1rt5V71.net
>>998
そういう分類は無意味だと思うけど、一応その考え方に沿うと、
きっかけとか媒体とか時とか。

990 :名無しさん@英語勉強中:2015/12/21(月) 05:40:58.48 ID:mkTskf16.net
君ら英語英語っていってるけど日本語は大丈夫かね?

俺は白井だよ
俺が白井だよ

上の文章の、は、と、が、の違いを何もしらない人が納得できるように説明できる?
みんな無意識に使い分けてるけど実際に言語化して他人に説明できる水準にあるひとはどれだけいる?

991 :名無しさん@英語勉強中:2015/12/21(月) 06:03:17.31 ID:ZuB4LnNy.net
>>990
それはあなたも含めて世界中で誰もうまく説明できないよw

992 :えワ:2015/12/21(月) 06:27:13.29 ID:51olIPwu.net
基準は個人それぞれだろうね。
ただ、「意味の強さ」は違うんで、それさえ理解してればいいだろうね。

I am Usui.
It' me, Usui.
I'm Usui, you are talking about.
必要なら、こんな感じで訳し分けるだろう。
いずれ、「俺は」より「俺が」の方が、俺に対する「意味が強い」という意識があればいいんじゃない。

寿司は好きだ。 ← 寿司の他にも好きな物がありそう。(外人なら、寿司が食べられる)
寿司が好きだ。 ← 寿司が一番好きな食べ物みたい。(外人なら、寿司が食べたい)

「細けぇ事はいいんだよ。臼井さんが田中さんにならなきゃ、通じるんだョ。」かな。

993 :名無しさん@英語勉強中:2015/12/21(月) 06:31:18.17 ID:ZuB4LnNy.net
いや990本人ができないから聞いてるわけで・・w

994 :名無しさん@英語勉強中:2015/12/21(月) 06:48:00.38 ID:hbudJAlL.net
はは後ろが新情報で、がは前が新情報というだけのこと。

995 :えワ:2015/12/21(月) 06:48:58.45 ID:51olIPwu.net
梅がてら。
「七人の侍」の最後、三船敏郎が盗賊の頭領に、撃たれながらも切りかかる時、
日本語は「貴様〜」で英語は「You」で、で You でいいかという話になった。
もっと強く、Fucking you とか、Son of a bitch とか、
つまり、「あなた」と「貴様」の明らかに日本語では意味が違う単語をどうする?っていうんでね。

「貴様と俺」なら You and I は「あなたと私」だから、brother か 表現変えないとみたいなはなし。
で、結局、映画を見てる人間は、あそこは、you と聞いてもきっと、脳内変換してるだろう。

>>993
そこは Situationic Linguistic の領分でね。
あなたにとっての「あなたは私」、私にとっての「あなたはあなた」になる。
"you means I" and "you means you"
リンゴを指さして「それ」と言えばリンゴ、レモンを指さして「それ」と言えばレモン、
"It" means the apple, the lemon and eveything.
「指さすという行為」が「それ」の意味を決める。

言語はコミュニケーションの道具で、机の上で考えるものではないんだよ。
もっと、外に出て、生身の人間と会話をする事だね。

996 :名無しさん@英語勉強中:2015/12/21(月) 06:50:19.78 ID:hbudJAlL.net
俺は白井だよ➡俺について新情報を言うと、白井という名前だよ
俺が白井だよ➡白井という名前が話題に出てるが、それについて新情報を言うと、俺のことだよ

997 :えワ:2015/12/21(月) 07:02:50.59 ID:51olIPwu.net
I am ARAI, everybody is talking.
He is Arai nobody knows.
同姓だけど、みんなが言ってるのは俺の事で、こいつじゃない。

俺が荒井、こいつは新井。
日本人の同姓異字も厄介なんだよね。
「同じだけど違う」っていう漢字の話をすると「深み」にはまっちゃうんだよね。
rough Arai と new Arai も変だしね。
まぁ、教科書にはない話だ。

998 :名無しさん@英語勉強中:2015/12/21(月) 07:15:10.17 ID:51olIPwu.net
♪ ∧,_∧  ♪
   ( ´・ω・) ))
 (( ( つ ヽ、   ♪ クリスマス〜
   〉 とノ )))
  (__ノ^(_)

999 :名無しさん@英語勉強中:2015/12/21(月) 07:15:44.63 ID:51olIPwu.net
   ∧_,∧ ♪
  (( (・ω・` )
♪  / ⊂ ) )) ♪ クリスマス〜
  ((( ヽつ 〈
   (_)^ヽ__)

1000 :えワ:2015/12/21(月) 07:16:35.20 ID:51olIPwu.net
ぱよ
♪ ∧,_∧  ♪
   ( ´・ω・) ))
 (( ( つ ヽ、   ♪
   〉 とノ )))
  (__ノ^(_)

ぱよ
   ∧_,∧ ♪
  (( (・ω・` )
♪  / ⊂ ) )) ♪
  ((( ヽつ 〈
   (_)^ヽ__)

ち〜ん!
♪ ∧,_∧
   (´・ω・`) ))
 (( (m9  9m   ♪
   〉    )))
  (__ノ^(_)

1001 :2ch.net投稿限界:Over 1000 Thread
2ch.netからのレス数が1000に到達しました。

総レス数 1001
393 KB
掲示板に戻る 全部 前100 次100 最新50
read.cgi ver.24052200